Está en la página 1de 121

CORPORACION UNIVERSITARIA DEL HUILA

CORHUILA

TRADUCCION DEL LIBRO


“FUNDAMENTALS OF BIOMECHANICS”

TITO FERNANDO MORENO GUTIERREZ


CODIGO: 1718119917

PRESENTANDO A:
RUTHBER RODRIGUEZ SERREZUELA

CORPORACIÓN UNIVERSITARIA DEL HUILA - CORHUILA


BIOMECANICA
NEIVA-HUILA
2020
CORPORACION UNIVERSITARIA DEL HUILA
CORHUILA

Capítulo 4
Estática: Sistemas en Equilibrio
4.1 Resumen
4.2 Leyes de Newton de la mecánica
4.3 Condiciones para el equilibrio
4.4 Diagramas de cuerpo libre
4.5 Procedimiento para analizar sistemas en equilibrio
4.6 Notas sobre las ecuaciones de equilibrio
4.7 Limitaciones y reacciones
4.8 Estructuras de soporte simple
4.9 Sistemas de poleas de cable y dispositivos de tracción
4.10 Estructuras incorporadas
4.11 Sistemas que implican fricción
4.12 Determinación del centro de gravedad
4.13 Problemas de ejercicio
CORPORACION UNIVERSITARIA DEL HUILA
CORHUILA

4.1 Resumen
La estática es un área dentro del campo de la mecánica aplicada, que se ocupa del análisis
de los cuerpos rígidos en equilibrio. En mecánica, el término equilibrio implica que el
cuerpo en cuestión está en reposo o en movimiento con una velocidad constante. Un
cuerpo rígido es aquel que se supone que sufre una no deformación bajo el efecto de
fuerzas aplicadas externamente. En realidad, no hay un material rígido y el concepto es una
aproximación. Para algunas aplicaciones, la extensión de las deformaciones implicadas
puede ser tan pequeña que la inclusión de las características de deformación del material
puede no influir en el análisis deseado. En tales casos, el material puede tratarse como un
cuerpo rígido.
Cabe señalar que, aunque el campo de la estática se ocupa de los análisis de los cuerpos
rígidos en equilibrio, los análisis que se ocupan de las características de deformación y
resistencia de los materiales también comienzan con el análisis estático. En este capítulo,
los principios se introducirá la estática y se proporcionarán las aplicaciones de estos
principios a sistemas relativamente sencillos. En el capítulo 5, las aplicaciones de los
mismos principios para analizar las fuerzas involucradas en y alrededor de las
principales articulaciones del cuerpo humano se construirán.
4.2 Las leyes de la mecánica de Newton
Toda la estructura de la mecánica se basa en unas pocas leyes básicas que
fueron establecidas por Sir Isaac Newton. La primera ley de Newton, la ley de inercia
establece que un cuerpo que originalmente está en reposo permanecerá en reposo, o un
cuerpo que se mueve con una velocidad constante en línea recta mantendrá su movimiento
a menos que una fuerza externa resultante no nula actúe sobre el cuerpo. La primera ley de
Newton debe ser considerada en conjunto con su segunda ley.
La segunda ley de Newton, la ley de aceleración establece que, si la fuerza neta o la
fuerza resultante que actúa sobre un cuerpo no es cero, entonces el cuerpo se acelerará en
la dirección de la fuerza resultante. Además, la magnitud de la aceleración del cuerpo
será directamente proporcional a la magnitud de la fuerza neta que actúa sobre el cuerpo e
inversamente proporcional a su masa. La segunda ley de Newton puede ser formulada
como:
F=ma (4.1)
Esto también se conoce como la ecuación del movimiento. En la Ec. (4.1), F es la red o la
fuerza resultante (suma vectorial de todas las fuerzas) que actúa sobre el cuerpo, m es la
masa del cuerpo, y a es su aceleración. Obsérvese que tanto la fuerza como la aceleración
son cantidades vectoriales mientras que la masa es una cantidad escalar. La masa es una
medida cuantitativa de la inercia de un cuerpo. La inercia se define como la tendencia de
un cuerpo a mantener su estado de reposo o movimiento uniforme a lo
CORPORACION UNIVERSITARIA DEL HUILA
CORHUILA

largo de una línea recta. La inercia también puede definirse como la resistencia a los
cambios en el movimiento de
un cuerpo. Cuanta más inercia tiene un cuerpo, más difícil es empezar a moverlo
desde el reposo, cambiar su movimiento o cambiar su dirección de movimiento.
La ecuación (4.1) es válida para los análisis de movimiento traslacional o lineal como los
que se realizan a lo largo de una trayectoria recta. Como se ilustra en la Fig. 4.1a, el
movimiento lineal se produce si todas las partes de un cuerpo se mueven a la misma
distancia al mismo tiempo y en la misma dirección. Un ejemplo típico de movimiento
lineal es el movimiento vertical de un ascensor en un hueco.

Fig. 4.1 a) Movimientos lineales y b) angulares


La ecuación (4.1) es sólo una forma de formular la segunda ley de Newton que
puede formularse alternativamente para el análisis del movimiento rotacional o
angular como:
M= I α(4.2)
En la Ec. (4.2), M es el momento neto o torque que actúa sobre el cuerpo, I es el momento
de inercia de la masa del cuerpo, y α es su aceleración angular. Como se ilustra en la Fig.
4.1b, el movimiento angular se produce cuando un cuerpo se mueve en una trayectoria
circular de manera que todas las partes del cuerpo se mueven en la misma dirección a
través del mismo ángulo al mismo tiempo.

Fig. 4.2 Martilleo


La tercera ley de Newton, la ley de acción-reacción, se basa en la observación de
que siempre hay dos lados cuando se trata de fuerzas. Una fuerza aplicada a un objeto
siempre
CORPORACION UNIVERSITARIA DEL HUILA
CORHUILA

es aplicada por otro objeto. Cuando un trabajador empuja una carreta, un niño tira de
una carreta, y un martillo golpea un clavo, la fuerza es aplicada por un cuerpo sobre
otro. La tercera ley de Newton establece que, si dos cuerpos están en contacto y el
cuerpo 1 está ejerciendo una fuerza sobre el cuerpo 2, entonces el cuerpo 2 aplicará una
fuerza sobre el cuerpo 1 de tal manera que las dos fuerzas tendrán una magnitud igual,
pero direcciones opuestas y actuarán en la misma línea de acción.
Por ejemplo, consideremos el caso de un martillo que empuja un clavo (Fig. 4.2). Es la
fuerza aplicada por el martillo sobre el clavo la que hace que éste avance hacia la
pared. Sin embargo, es la fuerza del clavo aplicada de nuevo en el martillo lo que hace que
éste se detenga después de cada impacto en el clavo.
Si presiona su mano contra el borde de un escritorio, puede ver cómo cambia la forma de
su mano, y también sentir la fuerza que ejerce el escritorio sobre su mano (Fig. 4.3).
Cuanto más fuerte se presione la mano contra el escritorio, más fuerte será la presión del
escritorio sobre la mano.

Fig. 4.3 Cuanto más fuerte empujes, más fuerte serás empujado.
Tal vez el mejor ejemplo que puede ayudarnos a entender la tercera ley de Newton es un
patinador que aplica una fuerza en una pared (Fig. 4.4). Al empujar contra la pared, el
patinador puede moverse hacia atrás. Es la fuerza ejercida por la pared hacia atrás sobre
el patinador lo que causa el movimiento del patinador.

Fig. 4.4 Un patinador sobre hielo


La tercera ley de Newton puede resumirse como "a cada acción le corresponde una
reacción igual y opuesta". Esta ley es particularmente útil para analizar problemas
complejos de mecánica y biomecánica en los que hay varios cuerpos que
interactúan.
4.3 Condiciones para el equilibrio
De acuerdo con la segunda ley de Newton, formulada en las Ecs. (4.1) y (4.2), un cuerpo
sometido a un movimiento de traslación o rotación tendrá aceleraciones lineales
CORPORACION UNIVERSITARIA DEL HUILA
CORHUILA

o angulares si la fuerza o el momento netos que actúa sobre el cuerpo no son cero. Si la
fuerza o el momento netos son cero, entonces la aceleración (lineal o angular) del cuerpo
es cero y, por consiguiente, la velocidad (lineal o angular) del cuerpo es constante o nula.
Cuando la aceleración es cero, se dice que el cuerpo está en equilibrio. Si la velocidad
también es cero, entonces el cuerpo está en equilibrio estático o en reposo.
Por lo tanto, hay dos condiciones que deben ser satisfechas para que un cuerpo esté
en equilibrio. Se dice que el cuerpo está en equilibrio traslacional si la fuerza neta
(suma vectorial de todas las fuerzas) que actúa sobre él es cero:
∑ 𝐹 = 0 (4.3)
El cuerpo está en equilibrio rotacional si el momento neto (suma vectorial de todos los
momentos) que actúa sobre él es cero:
∑ 𝑀 = 0 (4.4)
Obsérvese que puede haber varias fuerzas que actúan sobre un cuerpo. Por
ejemplo, consideremos el sistema de fuerza coplanaria de la Fig. 4.5. Supongamos que las
tres fuerzas actúan sobre un objeto en el plano xy. Estas fuerzas pueden expresarse en
términos de sus componentes a lo largo de las direcciones x e y:

Fig. 4.5 Un sistema de fuerza coplanaria en el xy-plano.


𝐹1 = 𝐹1𝑥𝑖 + 𝐹1𝑦𝑗
𝐹2 = −𝐹2𝑥 𝑖 + 𝐹2𝑦 𝑗 (4.5)
𝐹3 = −𝐹3𝑥 𝑖 + 𝐹3𝑦 𝑗

En la Ec.(4.5), F1x, F2x y F3x son los componentes escalares de F1, F2 y F3 en la


dirección x, x, F1y, F2yson sus componentes escalares en la dirección, i y j son
los vectores unitarios que indican las direcciones positivas de x y y. Estas ecuaciones
pueden sustituirse en la Ec. (4.3), y las componentes x e y de todas las fuerzas pueden
agruparse para escribir:

∑ 𝐹 = (𝐹1𝑥 − 𝐹2𝑥 − 𝐹3 𝑥)𝑖 + (𝐹1𝑦 + 𝐹2𝑦 − 𝐹3𝑥 )𝑗 = 0

Para que este equilibrio se mantenga, cada grupo debe ser individualmente igual a cero.
Eso es:
CORPORACION UNIVERSITARIA DEL HUILA
CORHUILA

𝐹1𝑥 − 𝐹2𝑥 − 𝐹3𝑥 = 0 (4.6) 𝐹1𝑦 + 𝐹2𝑦 − 𝐹3𝑦 = 0


En otras palabras, para que un cuerpo esté en equilibrio traslacional, la fuerza neta
que actúa en las direcciones x e y debe ser igual a cero. Para los sistemas de fuerzas
tridimensionales, la fuerza neta en la dirección z también debe ser igual a cero. Estos
resultados pueden resumirse de la siguiente manera:

∑𝐹𝑥 = 0

∑𝐹𝑦 = 0

∑𝐹𝑧 = 0

(4.7)

Precaución. Las condiciones de equilibrio dadas en la Ec. (4.7) son en forma escalar
y deben ser manejadas adecuadamente. Por ejemplo, al aplicar la condición de
equilibrio traslacional en la dirección x, las fuerzas que actúan en la dirección
positiva x deben sumarse y las fuerzas que actúan en la dirección negativa x deben
restarse.
Como ya se ha dicho, para que un cuerpo esté en equilibrio, tiene que estar tanto
en equilibrio traslacional como en equilibrio rotacional. Para que un cuerpo esté en
equilibrio rotacional, el momento neto sobre cualquier punto debe ser cero. Considere el
sistema coplanario de fuerzas ilustrado en la Fig. 4.6. Supongamos que estas fuerzas están
actuando sobre un objeto en el plano xy. Dejemos d1, d2 y d3, ser el momento
de las fuerzas F1, F2 y F3,relativo al punto O. Por lo tanto, los momentos debidos a
estas fuerzas sobre el punto O son:

Fig. 4.6 Momentos debidos a las fuerzas coplanarias

En la Ec. (4.8), k es el vector unitario que indica la dirección positiva de z. Momento


M1está actuando en la dirección positiva z mientras que M2 y M3 está actuando en la
CORPORACION UNIVERSITARIA DEL HUILA
CORHUILA

dirección negativa z. La Ecuación (4.8) puede ser sustituida en la condición de


equilibrio rotacional en la Ec. (4.4) para obtener:

Para que este equilibrio se mantenga:

Es decir, en este caso, el momento resultante en la dirección z debe ser cero. En lugar de
un sistema coplanario, si tuviéramos un sistema de fuerza tridimensional, entonces
podríamos tener momentos en las direcciones x e y también. La condición de equilibrio
rotacional también requeriría que tuviéramos un momento neto cero en las direcciones x
e y. Para los sistemas tridimensionales:

Precaución. Las condiciones de equilibrio rotacional dadas en la Ec. (4.10) también


están en forma escalar y deben ser manejadas adecuadamente. Por ejemplo, al aplicar
la condición de equilibrio rotacional en la dirección z, los momentos que actúan en la
dirección positiva z deben sumarse y los momentos que actúan en la dirección
negativa z deben restarse.
Obsérvese que para el sistema de fuerza coplanaria que se muestra en la Fig. 4.6, los
momentos pueden expresarse alternativamente como:

En otras palabras, con respecto al xy-plano, los momentos son en el sentido de las agujas
del reloj o en el sentido contrario. Dependiendo de la elección de la dirección positiva,
ya sea en el sentido de las agujas del reloj o en el sentido contrario, algunos de los
momentos son positivos y otros negativos. Si consideramos que los momentos en
sentido antihorario
son positivos, entonces el equilibrio rotacional sobre el punto O daría nuevamente la
Ec. (4.9):

4.4 Diagramas de cuerpo libre


CORPORACION UNIVERSITARIA DEL HUILA
CORHUILA

Los diagramas de cuerpo libre se construyen para ayudar a identificar las fuerzas
y momentos que actúan sobre las partes individuales de un sistema y para asegurar el uso
correcto de las ecuaciones de la estática. Para este propósito, las partes que constituyen un
sistema se aíslan de su entorno, y los efectos del entorno se reemplazan por las fuerzas y
momentos adecuados. En un diagrama de cuerpo libre se muestran todas las fuerzas y
momentos conocidos y desconocidos. Una fuerza es desconocida si no se conoce su
magnitud o dirección. Para las fuerzas conocidas, indicamos las direcciones correctas. Si
no se conoce la dirección de una fuerza, entonces tratamos de superponerla. Nuestra
predicción puede ser correcta o no. Sin embargo, la dirección correcta de la fuerza será un
resultado del análisis estático.
Por ejemplo, consideremos a una persona que intenta empujar un archivador a la derecha
en una superficie horizontal (Fig. 4.7). Hay tres partes que constituyen este sistema: la
persona, el archivador y la superficie horizontal que representa el suelo. Los diagramas de
cuerpo libre de estas partes se muestran en la Fig. 4.8. F es la magnitud de la fuerza
horizontal aplicada por la persona en el archivador para mover el archivador hacia la
derecha. Dado que las fuerzas de acción y reacción deben tener magnitudes iguales, F es
también la magnitud de la fuerza aplicada por el archivador sobre la persona. Como las
fuerzas de acción y reacción deben tener direcciones opuestas y la misma línea de acción,
la fuerza aplicada por el archivador sobre la persona tiende a empujar a la persona hacia la
izquierda. ��1 y ��2 son los pesos del archivador y de la persona, respectivamente, y
siempre están dirigidos verticalmente hacia abajo. ��1 �� ��2 son las magnitudes de
las fuerzas normales sobre la superficie horizontal aplicadas por el archivador y la
persona, respectivamente.
También son las magnitudes de las fuerzas aplicadas por la superficie horizontal en el
archivador y la persona. ��1 �� ��2 son las magnitudes de las fuerzas de fricción
��1 �� ��2 entre el archivador, la persona y la superficie horizontal, actuando ambos
en la dirección horizontal paralela a las superficies en contacto. Ya que el archivador
tiende a
moverse hacia la derecha, ��1 en el archivador actúa hacia la izquierda. Como la
persona tiende a moverse hacia la izquierda, ��2 en la persona actúa hacia la derecha
y es la fuerza que impulsa a la persona a empujar el archivador.

Para muchas aplicaciones, basta con considerar los diagramas de cuerpo libre de
sólo algunas de las partes individuales que forman un sistema. En el ejemplo
ilustrado en las Figs. 4.7 y 4.8, el diagrama de cuerpo libre del archivador
proporciona suficientes detalles para proceder a un análisis más detallado.

Fig. 4.7 Una persona empujando un archivador


CORPORACION UNIVERSITARIA DEL HUILA
CORHUILA

Fig. 4.8 Diagramas de cuerpo libre


4.5 Procedimiento para analizar los sistemas en equilibrio
El procedimiento general para analizar las fuerzas y momentos que actúan sobre
los cuerpos rígidos en equilibrio se describe a continuación.
- Dibuja un simple y nítido diagrama del sistema a analizar.
- Dibuja los diagramas de cuerpo libre de las partes que constituyen el sistema. Mostrar
todas las fuerzas y momentos conocidos y desconocidos en los diagramas de cuerpo
libre. Indique las direcciones correctas de las fuerzas y momentos conocidos. Si no se
conoce la dirección de una fuerza o un momento, intente predecir la dirección correcta,
indíquelo en el diagrama de cuerpo libre y no lo cambie en medio del análisis. La
dirección correcta de una fuerza o momento desconocido aparecerá en las soluciones.
Por ejemplo, un valor numérico positivo para una fuerza desconocida en la solución
indicará que se asumió la
dirección correcta. Un valor numérico negativo, por otra parte, indicará que la fuerza
tiene una dirección opuesta a la asumida.
En este caso, la magnitud de la fuerza debe hacerse positiva, debe indicarse la unidad
de fuerza adecuada y debe identificarse la dirección correcta en la solución.
- Adopte un sistema de coordenadas adecuado. Si se utiliza el sistema de
coordenadas rectangulares (x, y, z), se pueden orientar los ejes de coordenadas de
forma diferente para diferentes diagramas de cuerpo libre dentro de un mismo sistema.
Una buena orientación de los ejes de coordenadas puede ayudar a simplificar los análisis.
Resuelva todas las fuerzas y momentos en sus componentes, y exprese las fuerzas y
momentos en términos de sus componentes.
- Para cada diagrama de cuerpo libre, aplique las condiciones de equilibrio traslacional y
rotacional. Para los problemas tridimensionales, el número de ecuaciones disponibles es
de seis (tres traslacionales y tres rotacionales):
CORPORACION UNIVERSITARIA DEL HUILA
CORHUILA

Para los sistemas de fuerza bidimensionales en el plano xy-, el número de


ecuaciones disponibles es de tres (dos traslacionales y una rotacional, y las otras
tres se satisfacen automáticamente):

- Resuelve estas ecuaciones simultáneamente para las incógnitas. Incluya las


direcciones correctas y las unidades de fuerzas y momentos en la solución.
4.6 Notas relativas a las ecuaciones de equilibrio
- Recuerde que las ecuaciones de equilibrio dadas arriba están en forma "escalar" y no
tienen en cuenta las direcciones de las fuerzas y momentos involucrados. Por lo tanto,
por ejemplo, al aplicar la condición de equilibrio traslacional en la dirección x, dejemos
que una fuerza sea positiva si actúa en la dirección x positiva y dejemos que sea una
fuerza negativa si actúa en la dirección x negativa. Aplique esta regla en todas las
direcciones y también por momentos.
- Para los sistemas de fuerza coplanaria, mientras se aplica la condición de equilibrio
rotacional, elija un punto adecuado sobre el que se calculen todos los momentos. La
elección de este punto es arbitraria, pero una buena elección puede ayudar a
simplificar los cálculos.
- Para algunos problemas, puede que no sea necesario utilizar todas las condiciones
de equilibrio disponibles. Por ejemplo, si no hay fuerzas en la dirección x, entonces
la condición de equilibrio traslacional en la dirección x se satisface
automáticamente.
- A veces, puede ser más conveniente aplicar la condición de equilibrio rotacional más de
una vez. Por ejemplo, para un sistema de fuerzas coplanarias en el plano xy-, la condición
de equilibrio rotacional en la dirección z puede aplicarse dos veces considerando los
momentos de fuerzas sobre dos puntos diferentes. En tales casos, la tercera ecuación
independiente que podría utilizarse es la condición de equilibrio traslacional en la
dirección x o en la dirección y. La condición de equilibrio rotacional también puede
aplicarse tres veces en un solo problema. Sin embargo, los tres puntos sobre los que se
calculan los momentos no deben estar todos a lo largo de una línea recta.
CORPORACION UNIVERSITARIA DEL HUILA
CORHUILA

- Sistemas de dos fuerzas: Si sólo hay dos fuerzas que actúan sobre un cuerpo,
entonces el cuerpo estará en equilibrio si y sólo si las fuerzas son colineales, tienen
una magnitud igual y actúan en direcciones opuestas (Fig. 4.9).

Fig. 4.9 Un sistema colineal de dos fuerzas


- Sistemas de tres fuerzas: Si sólo hay tres fuerzas actuando sobre un cuerpo y si el cuerpo
está en equilibrio, entonces las fuerzas deben formar un sistema de fuerzas paralelo o
concurrente. Además, en cualquiera de los casos, las fuerzas forman un sistema de fuerzas
coplanarias. En la Fig. 4.10 se ilustra un sistema paralelo de tres fuerzas. Si se sabe que
dos de las tres fuerzas que actúan sobre un cuerpo son paralelas entre sí y el cuerpo está en
equilibrio, entonces podemos suponer fácilmente que la tercera fuerza es paralela a las
otras dos y establecer la línea de acción de la fuerza desconocida.

Fig. 4.10 Un sistema paralelo de tres fuerzas


En la Fig. 4.11 se ilustra un sistema concurrente de tres fuerzas. En este caso, las líneas
de acción de las fuerzas tienen un punto de intersección común. Si se conocen las líneas
de acción de dos de las tres fuerzas que actúan sobre un cuerpo y si no son paralelas y el
cuerpo está en equilibrio, entonces la línea de acción de la tercera fuerza puede
determinarse extendiendo las líneas de acción de las dos primeras fuerzas hasta que se
encuentren y trazando una línea recta que conecte el punto de intersección y el punto de
aplicación de la fuerza desconocida. La línea recta así obtenida representará la línea de
acción de la fuerza desconocida. Para analizar un sistema de fuerzas concurrentes,
las fuerzas que lo forman pueden trasladarse a lo largo de sus líneas de acción hasta el
punto de intersección (Fig. 4.11b). Las fuerzas pueden entonces resolverse en sus
componentes a lo largo de dos direcciones perpendiculares y las condiciones de equilibrio
traslacional pueden emplearse para determinar las incógnitas. En tales casos, la condición
de equilibrio rotacional se satisface automáticamente.
CORPORACION UNIVERSITARIA DEL HUILA
CORHUILA

Fig. 4.11 Un sistema concurrente de tres fuerzas


- Sistemas estáticamente determinados e indeterminados: Para los
problemas bidimensionales en estática, el número de ecuaciones de equilibrio disponibles
es de tres. Por lo tanto, el número máximo de incógnitas (fuerzas y/o momentos) que puede
determinarse aplicando las ecuaciones de equilibrio se limita a tres. Para los problemas
bidimensionales, si hay tres o menos incógnitas, entonces el problema es un uno
determinado estáticamente}. Si el número de incógnitas es superior a tres, entonces el
problema es estáticamente indeterminado. Esto significa que la información que tenemos a
través de la estática no es suficiente para resolver el problema completamente y que
necesitamos información adicional. Normalmente, la información adicional proviene de las
características materiales y propiedades de las partes que constituyen el sistema. La
extensión de este argumento para los problemas tridimensionales es sencilla. Como hay
seis ecuaciones de equilibrio, el número máximo de incógnitas que se pueden determinar
se limita a seis.
4.7 Limitaciones y reacciones
Una forma de clasificar las fuerzas es diciendo que son activas o reactivas. Las fuerzas
activas incluyen las cargas aplicadas externamente y las fuerzas gravitacionales. Las
fuerzas y momentos de restricción que actúan sobre un cuerpo se llaman reacciones. Por
lo general, las reacciones son desconocidas y deben determinarse aplicando las
ecuaciones de equilibrio.
Las fuerzas y momentos reactivos son los que ejerce el suelo, los elementos de apoyo
como rodillos, cuñas y bordes de cuchillos, y los miembros de conexión como cables,
pivotes y bisagras. El conocimiento de las características comunes de estos elementos de
apoyo y
miembros de conexión, que se resumen en el cuadro 4.1, puede ayudar a facilitar
la elaboración de diagramas de cuerpo libre y a resolver problemas particulares.
4.8 Estructuras de soporte simple
Los sistemas mecánicos están compuestos por una serie de elementos conectados
de diversas maneras. Los "rayos" constituyen los elementos fundamentales que forman los
bloques de construcción en la mecánica. Una viga conectada a otros elementos
CORPORACION UNIVERSITARIA DEL HUILA
CORHUILA

estructurales o al suelo por medios de rodillos, bordes de cuchillos, bisagras, conexiones


de pasadores, pivotes o cables forman un sistema mecánico. Analizaremos primero
ejemplos relativamente simples de tales casos para demostrar el uso de las ecuaciones de
equilibrio.
Tabla 4.1 Soportes, conexiones y elementos de conexión comúnmente
encontrados, y algunas de sus características .
CORPORACION UNIVERSITARIA DEL HUILA
CORHUILA

4.9 Sistemas de polea de cable y dispositivos de tracción


Las disposiciones de cable-polea se utilizan comúnmente para
elevar pesos y tienen aplicaciones en el diseño de dispositivos de
tracción utilizados en la rehabilitación de pacientes. Por ejemplo,
considere la disposición simple de la figura 4.21, en la que una
persona intenta levantar una carga mediante el uso de un sistema
de cable-polea. Suponga que la persona levantó la carga del piso
y la sostiene en equilibrio. El cable se enrolla alrededor de la
polea que está alojada en una caja que se fija al techo. La figura
4.22 muestra diagramas de cuerpo libre de la polea y la carga.
r es el radio de la polea y O representa un punto a lo largo de
la línea central (eje o eje) de la polea. Cuando la persona tira Fig. 4.21 A
del cable para levantar la carga, se aplica una fuerza sobre la disposición cable-
polea que se transmite al techo a través de la caja que aloja la polea.
polea. Como reacción, el techo aplica una fuerza sobre la polea a
través del eje que conecta la polea y la carcasa que aloja la polea.
En otras palabras, existe una fuerza de reacción R en la polea. En
la figura 4.22, la fuerza de reacción en el punto O está
representada por sus componentes escalares Rx y Ry. El cable se
enrolla alrededor de la polea entre los puntos A y B. Si ignoramos
los efectos de fricción entre el cable y la polea, entonces la
magnitud T de la tensión en el cable es constante en todas partes
del cable. Para probar este punto, supongamos que la magnitud
de la fuerza de tracción generada en el cable no es constante.
Sean TA y TB las magnitudes de las fuerzas de tracción
aplicadas por el cable sobre la polea en los puntos A y B,
respectivamente. Independientemente de la forma en que el

Fig. 4.22 Polea y carga.


cable se enrolle alrededor de la polea, el cable es tangente a la circunferencia de la polea en
los puntos de contacto inicial y final. Esto implica que las líneas rectas dibujadas desde el punto
O hacia los puntos A y B cortarán el cable en ángulo recto. Por lo tanto, medidos desde el
punto O, los brazos de momento de las fuerzas aplicadas por el cable sobre la polea son
siempre iguales al radio de la polea. Ahora, considere el equilibrio rotacional de la polea
sobre el punto O:
CORPORACION UNIVERSITARIA DEL HUILA
CORHUILA

Fig. 4.23 tracción de un


polea. Fig. 2.24 Tracción de tres
poleas.
Por lo tanto, la tensión T en el cable es la misma en los dos lados de la polea. Para el caso
ilustrado en las Figs. 4.21 y 4.22, el equilibrio vertical de la carga requiere que la tensión en
el cable sea igual al peso W de la carga a levantar.
Tenga en cuenta que Rx y Ry aplicados por el techo sobre la polea no producen ningún
momento alrededor de O. Si fuera necesario, estas fuerzas
podrían determinarse considerando el equilibrio horizontal y
vertical de la polea.

Las figuras 4.23 y 4.24 ilustran ejemplos de dispositivos de


tracción simples. Dichos dispositivos están diseñados para
mantener partes del cuerpo humano en posiciones particulares con
fines curativos. Para que tales dispositivos sean efectivos, deben
diseñarse para transmitir fuerzas adecuadamente a la parte del
cuerpo en términos de dirección y magnitud de la fuerza.
Diferentes disposiciones de cables y poleas pueden transmitir
diferentes magnitudes de fuerzas y en diferentes direcciones. Por
ejemplo, la tracción en la figura 4.23 aplica una fuerza horizontal
a la pierna con una magnitud igual al
peso en el platillo de pesas. Por otro lado, la tracción en la figura 4.24 aplica una fuerza
horizontal a la pierna con una magnitud dos veces mayor que
Fig. 2.25 Ejemplo 4.4
el peso en el platillo de pesas.

Ejemplo 4.4 Utilizando tres disposiciones diferentes de cable-polea que se muestran en la


figura 4.25, un bloque de peso W se eleva a una cierta altura. Para cada sistema, determine
cuánta fuerza se aplica a la persona que sostiene el cable.

Soluciones: Los diagramas de cuerpo libre necesarios para analizar cada sistema en la figura
4.25 se muestran en la figura 4.26. Para el análisis del sistema de la figura 4.25a, todo lo que
necesitamos es el diagrama de cuerpo libre del bloque (figura 4.26a). Para el equilibrio
vertical del bloque, la tensión en el cable debe ser T1 W. Por lo tanto, se aplica una fuerza
de magnitud igual al peso del bloque a la persona que sostiene el cable.
Para el análisis del sistema en la figura 4.25b, necesitamos examinar el diagrama de cuerpo
libre de la polea más cercana al bloque.
CORPORACION UNIVERSITARIA DEL HUILA
CORHUILA

(Figura 4.26b). Para el equilibrio vertical de la polea, la tensión


en el cable debe ser T2 W = 2. T2 es la tensión en el cable que
se enrolla alrededor de las dos poleas y es sostenido por la
persona. Por lo tanto, se aplica una fuerza con una magnitud
igual a la mitad del peso del bloque a la persona que sostiene el
cable.

Para el análisis del sistema en la figura 4.25c, nuevamente


necesitamos examinar el diagrama de cuerpo libre de la polea
más cercana al bloque (figura 4.26c). Para el equilibrio
vertical de la polea, la tensión en el cable debe ser T3 ¼ W = 3. T3 es la tensión
en el cable que envuelve las tres poleas y es sostenido por la persona. Por lo
Fig., 4.26 Diagramas tanto, se aplica una fuerza con una magnitud igual a un tercio del peso del
de cuerpo libre. bloque a la persona que sostiene el cable.

4.10 Estructuras integradas

La viga que se muestra en la figura 4.27 está empotrada o soldada


a una pared y se llama viga en voladizo. Las vigas en voladizo
pueden soportar momentos aplicados externamente así como
fuerzas. La viga de la figura 4.27 se suelda a la pared en el punto
A y se aplica una fuerza hacia abajo con magnitud P en el extremo
libre en el punto B. La longitud de la viga es l. En aras de la
simplicidad, ignore el peso de la viga. El diagrama de cuerpo libre
de la viga se muestra en la figura 4.28. En lugar de soldar, si la
viga estuviera articulada a la pared en el punto A, entonces, bajo Fig., 4.27 Viga
el efecto de la fuerza aplicada, la viga sufriría una rotación en el en voladizo.

sentido de las agujas del reloj. El hecho de que la viga no esté girando indica que hay
equilibrio rotacional. Este equilibrio rotacional se debe a un momento reactivo generado en
el extremo soldado de la viga. El momento en sentido antihorario con magnitud M en la
figura 4.28 representa este momento reactivo en el punto A.
Además, la fuerza aplicada tiende a trasladar la viga hacia abajo. Sin
embargo, el rayo no se está trasladando. Esto indica la presencia de una
fuerza de reacción ascendente en el punto A que equilibra el efecto de
la fuerza aplicada. La fuerza RA representa la fuerza reactiva en el
punto A. Dado que la viga está en equilibrio, podemos utilizar las
condiciones de equilibrio para determinar las reacciones. Por ejemplo,
considere el equilibrio de traslación del haz en la dirección y:

Fig., 4.28 Diagrama


de cuerpo libre de la
viga en voladizo.
CORPORACION UNIVERSITARIA DEL HUILA
CORHUILA

Ahora, considere el equilibrio rotacional de la viga alrededor del punto A y suponga que los
momentos en sentido antihorario son positivos:

Note que el momento reactivo con magnitud M es un “vector libre” que actúa en todas
partes a lo largo de la viga.

Fig. 4.29 Ejemplo 4.5

Ejemplo 4.5 Considere la viga horizontal uniforme que se muestra en la figura 4.29. La viga
se fija en el punto A y se aplica una fuerza que forma un ángulo β ¼ 60 ○ con la horizontal
en el punto B. La magnitud de la fuerza aplicada es P ¼ 100 N. El punto C es el centro de
gravedad de la viga. La viga pesa W ¼ 50 N y tiene una longitud l ¼ 2 m.

Fig. 4.30 Diagrama de


cuerpo libre de la viga.

Determine las reacciones generadas en el extremo fijo de la viga

Solución: El diagrama de cuerpo libre de la viga se muestra en la figura 4.30. Las direcciones
horizontal y vertical están indicadas por los ejes xey, respectivamente. Px y Py son los
componentes escalares de la fuerza aplicada P. Dado que conocemos magnitud y dirección de
P, podemos calcular fácilmente Px y Py:
CORPORACION UNIVERSITARIA DEL HUILA
CORHUILA

En la figura 4.30, la fuerza reactiva RA en el punto A se representa en términos de sus


componentes escalares RAx y RAy. No conocemos ni la magnitud ni la dirección de RA (dos
incógnitas). También tenemos un momento reactivo en el punto A con magnitud M que actúa
en una dirección perpendicular al plano xy.

En este problema, tenemos tres incógnitas: RAx, RAy y M. Para resolver el problema,
necesitamos aplicar tres condiciones de equilibrio. Primero, considere el equilibrio de
traslación del haz en la dirección x:

A continuación, considere el equilibrio de traslación de la viga en el dirección y:

Una vez que se determinan la magnitud y la dirección de las componentes de la fuerza de


reacción RA, podemos determinar la magnitud y la dirección de la fuerza de reacción en el
punto A:

Finalmente, considere el equilibrio rotacional de la viga alrededor del punto A. Suponiendo


que los momentos en sentido antihorario son positivos:
CORPORACION UNIVERSITARIA DEL HUILA
CORHUILA

Fig. 4.31 Fuerzas y


momento que
Como determinamos la magnitud de la fuerza de reacción y la actúan sobre la
magnitud y la dirección del momento reactivo en y punto A, también viga.
podemos determinar la dirección de la fuerza de reacción RA Si α es
un ángulo que la línea de acción de la reacción x P fuerza RA hace
con la horizontal, entonces:

El diagrama de cuerpo libre final de la viga se muestra en la figura


4.31.

Ejemplo 4.6 Considere la viga en forma de L que se ilustra en la figura


4.32. La viga está soldada a la pared en el punto A, el brazo AB se
extiende en la dirección z positiva y el brazo BC se extiende en la
dirección x negativa. Se aplica una fuerza P en la dirección y negativa
en el punto B. Las longitudes de los brazos AB y BC son a ¼ 20 cm
yb ¼ 30 cm, respectivamente, y la magnitud de la fuerza aplicada es
P ¼ 120 N.
Suponiendo que el peso de la viga es insignificante en comparación
con la magnitud de la fuerza aplicada, determine las reacciones en el
extremo fijo de la viga.
Este es un problema tridimensional y emplearemos dos métodos para
analizarlo. El primer método utilizará los conceptos de par y momento
de par presentados en el capítulo anterior, y el segundo método
utilizará las propiedades vectoriales de fuerzas y momentos.

Solución A: método escalar El método escalar para analizar el problema se describe en la


figura 4.33 e implica la traslación de la fuerza aplicada sobre la viga del punto C al punto
A. Primero, para trasladar P del punto C al punto B, coloque un
Fig. 4.32 Ejemplo 4.6
par de fuerzas en el punto B que son iguales en magnitud (P) y
que actúan en direcciones opuestas, de modo que la línea de
acción común de las nuevas fuerzas es paralela a la línea de

acción de la fuerza original en el punto C (figura 4.33a ). La fuerza hacia abajo en el punto
CORPORACION UNIVERSITARIA DEL HUILA
CORHUILA

C y la fuerza hacia arriba en el punto B forman un par. Por lo tanto,


pueden reemplazarse por un momento de par ilustrado por una flecha
de dos puntas en la figura 4.33b. La magnitud del momento de par
es M1 ¼ bP. Aplicando la regla de la mano derecha, podemos ver
que el par-momento actúa en la dirección z positiva. Por lo tanto:

Fig. 4.33
Método escalar

Como se ilustra en la figura 4.33c, para trasladar la fuerza del punto B al punto A, coloque otro
par de fuerzas en el punto A con igual magnitud y direcciones opuestas. Esta vez, la fuerza
hacia abajo en el punto B y la fuerza hacia arriba en el punto A forman un par y, nuevamente,
pueden reemplazarse por un momento de par (figura 4.33d). La magnitud de este momento
de par es M2 ¼ aP y actúa en la dirección x positiva. Por lo tanto:

La figura 4.34 muestra el diagrama de cuerpo libre de la viga. P es la magnitud de la fuerza


aplicada en el punto C que se traslada al punto A, y M1 y M2 son las magnitudes de los
pares-momentos. RAx, RAy y RAz son las componentes escalares de
la fuerza reactiva en el punto A, y MAx, MAy y MAz son las
componentes escalares del momento reactivo en el punto A. Considere
el equilibrio de traslación del haz en la dirección x:

El equilibrio de traslación del haz en la dirección y requiere que:

Para el equilibrio de traslación del haz en la dirección z: Fig. 4.34 Diagrama


de cuerpo libre de
la viga.

Por lo tanto, solo hay una componente distinta de cero de la fuerza reactiva sobre la viga en
el punto A y actúa en la dirección y positiva. Ahora, considere el equilibrio rotacional de la
viga en la dirección x:
CORPORACION UNIVERSITARIA DEL HUILA
CORHUILA

Para el equilibrio rotacional de la viga en la dirección y:

Finalmente, el equilibrio rotacional de la viga en la dirección z


requiere que:

Por lo tanto, el momento reactivo en el punto A tiene dos componentes distintos de cero
en las direcciones xey. Ahora que determinamos las componentes de la fuerza reactiva y
el momento en el punto A, también podemos calcular las magnitudes de la fuerza
Fig. 4.35 y el momento resultantes en el punto A:
Método
vectorial.

Solución B: Método vectorial El segundo método para analizar el mismo problema utiliza
las propiedades vectoriales de los parámetros involucrados. Por ejemplo, la fuerza
aplicada en el punto C y el vector de posición del punto C en relación con el punto A se
pueden expresar como (Fig. 4.35):
CORPORACION UNIVERSITARIA DEL HUILA
CORHUILA

Aquí, i, j y k son vectores unitarios que indican x, y, y direcciones z , respectivamente. El


diagrama de cuerpo libre de la viga se muestra en la figura 4.36 donde las fuerzas reactivas y
los momentos están representados por sus componentes escalares de manera que:

Fig. 4.36 Diagrama


de cuerpo libre de
la viga.

Primero, considere el equilibrio de traslación de la viga:

Para que este equilibrio se mantenga:


CORPORACION UNIVERSITARIA DEL HUILA
CORHUILA

Como se discutió en el capítulo anterior, por definición, el momento es el producto cruzado


(vector) de los vectores de posición y fuerza. Por lo tanto, el momento MC relativo al punto
A debido a la fuerza P.

Aplicado en el punto C es:

Ahora considere el equilibrio rotacional de la viga sobre el punto A:

Para que este equilibrio se mantenga:

Todos estos resultados son consistentes con los obtenidos mediante el método de análisis
escalar.

Observaciones
CORPORACION UNIVERSITARIA DEL HUILA
CORHUILA

• Analizamos este problema de dos formas. Está claro que el


método de análisis escalar requiere manipulaciones
matemáticas menos rigurosas. Sin embargo, tiene sus
limitaciones. Por ejemplo, en este ejemplo, la fuerza aplicada
en el punto C tenía solo un componente distinto de cero. ¿Qué
pasaría si tuviéramos una fuerza en el punto C con
componentes distintos de cero en las direcciones xyz así como
en la dirección y? El método escalar aún podría aplicarse paso
a paso considerando un componente de la fuerza aplicada a la
vez, resolviendo el problema para ese componente solo,
repitiendo esto para los tres componentes y luego
superponiendo las tres soluciones para obtener la solución Fig. 4.37
definitiva. Obviamente, esto llevaría bastante tiempo. Por otro Incluyendo los
lado, la extensión del método vectorial para analizar tales pesos de los
brazos.
problemas es muy sencilla. Todo lo que se necesita hacer es redefinir el vector de
fuerza aplicada como P ¼ Pxi þ Pyj þ Pzk, y

Realice exactamente el mismo procedimiento descrito anteriormente.

• En este ejemplo, se indica que el peso de la viga era insignificante en comparación con la
fuerza aplicada en el punto C. ¿Qué pasaría si el peso de la viga no fuera despreciable?
Como se ilustra en la figura 4.37, suponga que conocemos los pesos W1 y W2 de los
brazos AB y BC de la viga junto con sus centros de gravedad (puntos D y E). Además,
suponga que D está ubicado en el medio del brazo AB y E es ubicado en el medio del brazo
BC. Ya hemos discutido la representación vectorial de RA y P. Las representaciones
vectoriales de W1 y W2 son:

Los vectores de posiciones de los puntos D y E con respecto al punto A son:

Por tanto, los momentos debidos a W1 y W2 relativos al punto A son:


CORPORACION UNIVERSITARIA DEL HUILA
CORHUILA

El equilibrio de traslación de la viga produciría:

El equilibrio rotacional de la viga produciría:

• Tenga en cuenta las similitudes entre este y el ejemplo del hombro del capítulo anterior
(ejemplo 3.5).

4.11 Sistemas que involucran fricción

Las fuerzas de fricción se analizaron en detalle en el cap. 2. Aquí


analizaremos un problema en el que las fuerzas de fricción juegan
un papel importante.

Ejemplo 4.7 La figura 4.38 ilustra a una persona que intenta


empujar un bloque hacia arriba de una superficie inclinada
Fig. 4.38 Ejemplo 4.7 aplicando una fuerza paralela a la pendiente. El peso del
CORPORACION UNIVERSITARIA DEL HUILA
CORHUILA

bloque es W, el coeficiente de fricción máxima entre el bloque y la pendiente es μ, y la


pendiente forma un ángulo θ con la horizontal.

Determine la magnitud P de la fuerza mínima que debe aplicar la persona para superar los
efectos de fricción y gravitación para comenzar a mover el bloque en términos de W, μ y θ.

Solución: tenga en cuenta que si la persona empuja el bloque aplicando una fuerza más
cerca de la parte superior del bloque, el bloque puede inclinarse (girar en el sentido de las
agujas del reloj) sobre su borde inferior derecho. Aquí, asumiremos que no existe tal efecto
y que la superficie inferior del bloque permanece en pleno contacto con el suelo. El
diagrama de cuerpo libre del bloque se muestra en la figura 4.39. xey corresponden a las
direcciones paralelas y perpendiculares a la pendiente, respectivamente. P es la magnitud de
la fuerza aplicada por la persona sobre el bloque en la dirección x, f es la fricción

fuerza aplicada por la superficie inclinada sobre el bloque en la dirección x negativa, N es la


fuerza normal aplicada por la superficie inclinada sobre el bloque en la dirección y positiva,
y W es el peso del bloque que actúa verticalmente hacia abajo. El peso del bloque tiene
componentes en las direcciones xey que se pueden determinar a partir de la geometría del
problema (ver Fig. A.3 en el Apéndice A):

Dado que la dirección de movimiento prevista del bloque es en la dirección x positiva, la


fuerza de fricción sobre el bloque actúa en la dirección x negativa tratando de evitar que el
bloque suba por la pendiente. La magnitud f de la fuerza de fricción es directamente
proporcional a la magnitud N de la fuerza normal aplicada por la superficie inclinada sobre
el bloque y el coeficiente de fricción, μ, es la constante de proporcionalidad. Por lo tanto, f,
N y μ están relacionados mediante:

Las incógnitas en este ejemplo son P y N. Dado que tenemos una expresión
que relaciona f y N, si se conoce N, también lo es f. Para la solución del
problema, primero considere el equilibrio del bloque en la dirección y:
CORPORACION UNIVERSITARIA DEL HUILA
CORHUILA

Fig. 4.39 Diagrama de


cuerpo libre del
Una vez que se determina N, entonces: bloque.

Se nos pide que determinemos la fuerza mínima que debe aplicar la persona para superar los
efectos de fricción y gravitación para comenzar a mover el bloque. Hay equilibrio en el
instante justo antes comienza el movimiento. Por tanto, podemos aplicar la condición de
equilibrio en la dirección x:

Sustituir Ecuación. (i) y (ii) en la ecuación. (iii) junto con Wx = W sen θ:

Ésta es una solución general para P en términos de W, μ y θ. Esta solución es válida para
cualquier θ menor que 90 ○, incluido θ ¼ 0 ○, que representa una superficie horizontal plana
(figura 4.40). Para θ ¼ 0 ○, sen 0 ¼ 0 ○ y cos 0 ¼ 0 ○. Por tanto, la fuerza necesaria para
empezar a mover el mismo bloque sobre una superficie horizontal es:

Para tener un ejemplo numérico, suponga que W = 1000 N, μ = 0: 3 y θ = 15 ○. Luego:

Por lo tanto, para poder comenzar a mover un bloque de 1000 N


hacia arriba por la pendiente de 15 ○ que tiene un coeficiente de
fricción superficial de 0.3, la persona debe aplicar una fuerza
ligeramente mayor a 548.6 N en una dirección paralela a la
pendiente.
Para comenzar a mover el mismo bloque sobre una superficie
horizontal con el mismo coeficiente de fricción, la persona debe aplicar una
Fig. 4.40 Empujar un fuerza horizontal de:
bloque sobre una
superficie horizontal.
CORPORACION UNIVERSITARIA DEL HUILA
CORHUILA

En comparación con una superficie horizontal, la persona debe aplicar aproximadamente un


83% más de fuerza sobre el bloque para comenzar a mover el bloque en la pendiente de 15
○, que se calcula como:

4.12 Determinación del centro de gravedad


CORPORACION UNIVERSITARIA DEL HUILA
CORHUILA

Como se discutió en el Cap. 2, se puede considerar que cada objeto consta de un número
infinito de partículas sobre las que actúa la fuerza de gravedad, formando así un sistema de
fuerzas distribuidas. La resultante de estas fuerzas o pesos individuales de partículas es igual
al peso total del objeto, actuando como una carga concentrada en el centro de gravedad del
objeto. Un concepto relacionado con el centro de gravedad es el del centro de masa, que es
un punto en el que se supone que toda la masa de un objeto es concentrado. En general, existe
una diferencia entre los centros de masa y gravedad de un objeto. Puede valer la pena
considerar esto si el objeto es lo suficientemente grande como
para que la magnitud de la aceleración gravitacional varíe en Fig. 4.41 El centro de
diferentes partes del objeto. Para nuestras aplicaciones, los
gravedad del papel se
centros de masa y gravedad de un objeto se refieren al mismo
encuentra en la
punto. También existe el concepto de línea de gravedad que se
utiliza para referirse a la línea vertical que pasa por el centro de intersección de las líneas
gravedad. aa y bb (método de
suspensión).

El centro de gravedad de un objeto es tal que si el objeto se corta en dos partes por cualquier
plano vertical que pase por el centro de gravedad, entonces el peso de cada parte sería igual.
Por lo tanto, el objeto se puede equilibrar sobre un filo de cuchillo que se encuentra
directamente debajo de su centro de gravedad oa lo largo de su línea de gravedad. Si un objeto
tiene una geometría simétrica, bien definida y una composición uniforme, entonces su centro
de gravedad está ubicado en el centro geométrico del objeto. Existen varios métodos para
encontrar los centros de gravedad de objetos de forma irregular. Un método es "suspender"
el objeto. A modo de ilustración, considere la hoja de papel que se muestra en la figura 4.41.
El centro de gravedad del papel está ubicado en el punto C. Sean O y Q dos puntos en el papel.
Si el papel se fija a la pared en el punto O, habrá un momento neto en el sentido de las agujas
del reloj distinto de cero que actuará sobre el papel con respecto al punto O porque el centro de
gravedad del papel se encuentra a la derecha de O.Si el papel se suelta , el momento alrededor
de O debido al peso W del papel hará que el papel se balancee primero en el sentido de las
agujas del reloj, oscile y finalmente se detenga en una posición en la que C se encuentra
a lo largo de una línea vertical aa (línea de gravedad) que pasa por
punto O (figura 4.41b). En esta posición, el momento neto
alrededor del punto O es cero porque la longitud del brazo de
momento de W con relación al punto O es cero. Si el papel se
sujeta con alfileres en el punto Q, el papel se balanceará en el
sentido contrario a las agujas del reloj y pronto se detendrá en una
posición en la que el punto C se encuentra directamente debajo del
punto Q, oa lo largo de la línea vertical bb que pasa por el punto Q
( Figura 4.41c). El punto en el que las líneas aa y bb se cruzan
indicará el centro de gravedad del papel, que es el punto C.
CORPORACION UNIVERSITARIA DEL HUILA
CORHUILA

Tenga en cuenta que una hoja de papel es un objeto plano con un grosor insignificante y que
suspender el papel en dos puntos es suficiente para localizar su centro de gravedad. Para un
objeto tridimensional, el objeto debe estar suspendido
en tres puntos en dos planos diferentes. Fig. 4.42 Método
de equilibrio.
Otro método para encontrar el centro de gravedad es

“equilibrar” el objeto en el filo de un cuchillo. Como se ilustra en la figura 4.42, para


determinar el centro de gravedad de una persona, primero balancee una tabla en el borde del
cuchillo y luego coloque a la persona en decúbito supino sobre la tabla. Ajuste la posición de
la persona en la tabla hasta que la tabla esté nuevamente equilibrada (Fig. 4.42a). La distancia
horizontal entre los pies de la persona y el punto de contacto del filo de la navaja y la tabla
es la altura del centro de gravedad de
la persona.
Considere un plano que pasa por el punto de contacto del filo
de la navaja con la tabla que corta a la persona en porciones
superior e inferior. El centro de gravedad de la persona se
encuentra en algún lugar de este plano. Tenga en cuenta que el
centro de gravedad de un cuerpo tridimensional, como un ser
humano, tiene tres coordenadas. Por tanto, el mismo método
debe repetirse en otros dos planos para establecer el centro de
gravedad exacto. Para este propósito, considere el equilibrio
anteroposterior de la persona que producirá dos planos
adicionales como se ilustra en (Fig. 4.42b, c). La intersección
de estos planos corresponderá al centro de gravedad de la
persona.

El tercer método para encontrar el centro de gravedad de un


cuerpo implica el uso de una “tabla de reacción” con dos filos
fijos a su superficie inferior (Fig. 4.43). Suponga que se
conocen el peso WB de la tabla y la distancia l entre los
bordes de la cuchilla. Uno de los dos bordes (A) descansa sobre una plataforma y el otro
borde (B) descansa sobre una escala, de manera que el tablero quede horizontal. La ubicación
del centro de gravedad de una persona se puede
determinar colocando a la persona en el tablero y Fig. 4.43 Método de
registrando el peso indicado en la báscula, que es la placa de reacción.
esencialmente la magnitud RB de la fuerza de reacción en

el tablero en el punto B. La figura 4.43b ilustra el diagrama de cuerpo libre de la placa. RA


es la magnitud de la fuerza de reacción en el punto A, WP es el peso conocido de la persona.
El peso WB del tablero actúa en el centro geométrico (punto C) del tablero que es equidistante
de los puntos A y B. D es un punto en el tablero directamente debajo del centro de gravedad
de la persona. La distancia desconocida entre los puntos A y D se designa por xcg, que se
puede determinar considerando el equilibrio rotacional de la tabla sobre el punto A.
Suponiendo que los momentos en el sentido de las agujas del reloj son positivos:
CORPORACION UNIVERSITARIA DEL HUILA
CORHUILA

Resolver esta ecuación para xcg producirá:

Si la persona se coloca en la tabla de modo que los pies estén directamente sobre el filo del
cuchillo en el punto A, xcg designará la altura del centro de gravedad de la
persona medida desde el nivel del suelo.

A veces, debemos tratar con un sistema formado por partes con centros
de gravedad conocidos donde la tarea es determinar el centro de gravedad
del sistema en su conjunto. Esto se puede lograr simplemente utilizando
la definición del centro de gravedad. Considere el sistema que se muestra
en la figura 4.44, que se compone de tres esferas con pesos W1, W2 y W3
conectados entre sí a través de varillas. Suponga que los pesos de las
varillas son insignificantes. Sean x1, x2 y x3 las coordenadas x de los
Fig. 4.44. Xcg es el centros de gravedad de cada esfera. El momento neto sobre el punto O debido
componente X del a los pesos individuales de las esferas es:
centro de gravedad.

El peso total del sistema es W1, W2 y W3, que se supone que actúa en el centro
de gravedad del sistema como un todo. Si xcg es la coordenada x del centro de
gravedad de todo el sistema, entonces el momento del peso total del sistema
respecto al punto O es:

Fig. 4.45 Ycg es el Las dos últimas ecuaciones se pueden combinar para eliminar MO, lo que
componente Y del producirá:
centro de gravedad.
CORPORACION UNIVERSITARIA DEL HUILA
CORHUILA

Este resultado se puede generalizar para cualquier sistema


compuesto por
n partes:

Fig. 4.46 X es el centro


de gravedad del La ecuación (4.12) proporciona solo la coordenada x del centro de gravedad
sistema. del sistema. Para determinar el centro exacto, también se debe determinar la
coordenada y del centro de gravedad. Para este propósito, todo el sistema
debe girarse en un ángulo, preferiblemente 90 °, como se ilustra en la figura
4.45. Si y1, y2 e y3 corresponden a las coordenadas y de los centros de gravedad de las
esferas, entonces la coordenada y del centro de gravedad del sistema en su conjunto es:

Para cualquier sistema compuesto por n partes:

En la figura 4.46, el centro de gravedad de todo el sistema está ubicado


en el punto de intersección de las líneas perpendiculares que pasan por
xcg e ycg.

Ejemplo 4.8 Considere la pierna que se muestra en la figura 4.47, que


está flexionada en ángulo recto. Las coordenadas de los centros de
Fig. 4.47 Localizar gravedad de la pierna entre las articulaciones de la cadera y la rodilla (parte
el cetro de gravedad superior de la pierna), las articulaciones de la rodilla y el tobillo y el pie, medidas
de una pierna
flexionada.
CORPORACION UNIVERSITARIA DEL HUILA
CORHUILA

desde el nivel del suelo directamente en línea con la articulación de la cadera, se dan en la
Tabla 4.2. . Los pesos de los segmentos de la pierna como porcentajes del peso total W de la
persona también se proporcionan en la Tabla 4.2. Determina la ubicación del centro de
gravedad de toda la pierna.

Solución: Las coordenadas (xcg, ycg) del centro de gravedad de toda


PART X Y %W
la pierna se pueden determinar utilizando las ecuaciones. (4.12) y
(CM) (CM) (4.13). Usando la Ec. (4.12):
1 17.3 51.3 10.6
2 42.5 32.8 4.6
3 45.0 3.3 1.7
Tabla 4.2 Ejemplo 4.8

Para determinar la coordenada y del centro de gravedad de la pierna, debemos rotar la pierna
90 °, como se ilustra en la figura 4.48, y aplicar la ecuación. (4.13):

Por lo tanto, el centro de gravedad de toda la extremidad inferior cuando se flexiona en ángulo
recto se encuentra a una distancia horizontal de A 26,9 cm de la articulación de la cadera ya
una altura de 41,4 cm medida desde el nivel del suelo.

Observaciones

• Al estar de pie con las extremidades superiores rectas, el centro


de gravedad de una persona se encuentra dentro de la pelvis
anterior a la segunda vértebra sacra. Si el origen del sistema de
coordenadas rectangulares se coloca en el centro de gravedad de
la persona, entonces el plano xy corresponde al plano frontal,
coronal o longitudinal, el plano yz se llama plano sagital y el Fig. 4.48 X es el
plano xz -plano es el plano horizontal o centro de gravedad.
CORPORACION UNIVERSITARIA DEL HUILA
CORHUILA

transversal. El plano frontal divide el cuerpo en porciones delantera y trasera, el plano


sagital divide el cuerpo en porciones derecha e izquierda, y el plano horizontal lo divide
en porciones superior e inferior.
• El centro de gravedad de todo el cuerpo varía de persona a persona dependiendo de la
estructura. Para una persona dada, la posición del centro de gravedad puede cambiar
dependiendo de los cambios en la alineación relativa de las extremidades durante una
actividad física particular. La ubicación de los centros de gravedad de las extremidades
superiores e inferiores también puede variar. Por ejemplo, el centro de gravedad de una
extremidad inferior se desplaza hacia atrás cuando se flexiona la rodilla. El centro de
gravedad de todo el brazo se desplaza hacia adelante a medida que se flexiona el codo.
Estas observaciones sugieren que cuando la rodilla está flexionada, la pierna tenderá a
moverse hacia adelante llevando el centro de gravedad de la pierna directamente debajo
de la articulación de la cadera. Cuando se flexiona el codo, el brazo tenderá a moverse
hacia atrás llevando su centro de gravedad debajo de la articulación del hombro.

4.13 Problemas de ejercicio

Problema 4.1 Como se ilustra en la figura 4.12, considere una persona de pie sobre una viga
horizontal uniforme que descansa sobre soportes de rodillos y bordes de cuchillo sin fricción.
A y B son dos puntos de contacto entre la viga y la cuchilla y el soporte del rodillo,
respectivamente. El punto C es el centro de gravedad del rayo y es equidistante de los puntos
A y B. D es el punto del rayo directamente debajo del centro de gravedad de la persona.
Debido a los pesos de la viga y la persona, hay reacciones en la viga en los puntos A y B. Si
el peso de la persona es W ¼ 625 N y las reacciones en los puntos A y B son RA ¼ 579,4 N
y RB ¼ 735,6 N.

(a) Determine el peso (W) de la viga.


(b) Determine la longitud (l) de la viga.

Respuestas: (a) W = 690 N; (b) l = 4 m

Problema 4.2 Como se ilustra en la figura 4.49, considere una persona


de 80 kg que se prepara para zambullirse en una piscina. El trampolín
está representado por una viga horizontal uniforme que está articulada al
suelo en el punto A y sostenida por un rodillo sin fricción en el punto D.
B es un punto en el tablero directamente debajo del centro de gravedad
de la persona. La distancia entre los puntos A y B es l ¼ 6 my la distancia
entre los puntos A y D es d 2
m. (Tenga en cuenta que un tercio de la tabla se encuentra a la
izquierda del soporte del rodillo y dos tercios a la derecha. Por lo Fig. 4.49 Problema 4.2.
CORPORACION UNIVERSITARIA DEL HUILA
CORHUILA

tanto, por motivos de análisis de fuerza, se puede suponer que la tabla consta de dos tablas con
dos pesos diferentes conectada en el punto D.)
Si el trampolín tiene un peso total de 1500 N, determine las reacciones en la viga en los
puntos A y D.

Respuestas:

Problema 4.3 La viga horizontal uniforme que se muestra en la


figura 4.50 está articulada al suelo en el punto A y sostenida por
un rodillo sin fricción en el punto D. La distancia entre los puntos
A y B es l = 4 my la distancia entre los puntos A y D es d = 3 m.
En el punto B se aplica una fuerza que forma un ángulo β ¼ 60 ○
con la horizontal. La magnitud de la fuerza aplicada es P ¼ 1000
N. El peso total de la viga es W ¼ 400 N.
Al notar que tres cuartos de la viga están a la izquierda del soporte
del rodillo y un cuarto a la derecha, calcule las componentes xey Fig. 4.50 Problema
de las fuerzas de reacción sobre la viga en los puntos A y D. 4.3.

Respuestas:

Problema 4.4 La viga horizontal uniforme que se muestra en la figura


4.51 está articulada a la pared en el punto A y sostenida por un cable
unido a la viga en el punto C. El punto C también representa el centro
de gravedad de la viga. En el otro extremo, el cable se fija a la pared de
modo que forme un ángulo θ ¼ 68 ○ con la horizontal. Si la longitud de
la viga es l ¼ 4 my el peso de la viga es W 400 N, calcule la tensión T
en el cable y los componentes de la fuerza de reacción sobre la viga en Fig. 4.51 Problema
el punto A. 4.4.

Respuestas:
CORPORACION UNIVERSITARIA DEL HUILA
CORHUILA

Problema 4.5 Considere una estructura ilustrada en la figura 4.52. La estructura


incluye una viga horizontal con bisagras a la pared en el punto A y tres accesorios
eléctricos idénticos unidos a la viga en los puntos B, D y E con el punto B que
identifica el extremo libre de la viga. Además, las distancias entre los puntos de
fijación de los aparatos eléctricos son iguales entre sí (BD ¼ DE a b 35 cm). El
punto C identifica el centro de gravedad de la viga y es equidistante de los puntos
A y B. El peso de la viga es W = 230
W
N y cada aparato eléctrico pesa W1= W2 = W3 = 45 N. Además, se conecta un cable a la
viga en el punto B formando un ángulo de 45 con la horizontal. Fig. 4.52 Problema 4.5.
En otro extremo, el cable se une a la pared para mantener la viga en su
lugar. Si la longitud de la viga es l = 2,5 m.
(a) Determine la tensión (T) en el cable.
(b) Determine la magnitud de la fuerza de reacción (RA) en el punto A.

(c) Determine la tensión (T1) en el cable cuando forma un ángulo α= 65° con la horizontal.
(d) Determine el cambio en la magnitud de la fuerza de reacción en el punto A cuando el
cable forma un ángulo α = 65° con la horizontal.

Respuestas: a) T=817,2 N; (b) RA=615,7 N; (c) T1=634,8 N; (d) Disminución del


44,6%

Problema 4.6 Usando dos arreglos diferentes de cable-polea que se


muestran en la figura 4.53, un bloque de peso W se eleva a cierta altura. Para
cada sistema, determine cuánta fuerza se aplica a la persona que sostiene
el cable.

Respuestas: T1 =W / 2 T2 = W / 4
Fig. 4.53 Problema
4.6.
CORPORACION UNIVERSITARIA DEL HUILA
CORHUILA

Problema 4.7 Como se ilustra en la figura 4.53b, considere a una


persona que está tratando de elevar una carga a una cierta altura
usando un arreglo de cable-polea. Si la fuerza aplicada por la
persona sobre el cable es T = 65 N, determine la masa de la carga.
Respuesta: m = 26,5 kg
Problema 4.8 Usando una disposición de cable-polea que se
muestra en la figura 4.54, un bloque de masa m 50 kg se eleva
desde el suelo hasta una cierta altura. Determine la magnitud de la
fuerza T aplicada por el trabajador que realiza la tarea de elevación
en el cable.
Fig. 4.54 Problema
4.8.
Respuesta: T =163,

Problema 4.9 Considere el dispositivo de tracción Russel dividido


y un modelo mecánico de la pierna que se muestra en la figura
4.55. La pierna se mantiene en la posición mostrada por dos pesos
que están conectados a la pierna a través de dos cables. El peso
combinado de la pierna y el yeso es W = 300 N. l es la distancia
horizontal entre los puntos A y B donde los cables están unidos a
la pierna. El punto C es el centro de gravedad de la pierna, incluido
el yeso, que se encuentra a una distancia de dos tercios de l, medida
desde el punto A. El ángulo que forma el cable 2 con la horizontal
se mide como β ¼ 45 ○.
Determine las tensiones T1 y T2 en los cables, los pesos W1 y W2, Fig. 4.55 Problema
y el ángulo α que el cable 1 forma con la horizontal, de modo que la 4.9.
pierna permanezca en equilibrio en la posición que se muestra.

Respuestas: T1 =W1 = 223.6N T2 = W2 = 282.8N α = 26.6°

Problema 4.10 Considere la viga en voladizo horizontal uniforme que


se muestra en la figura 4.56. La viga está fija en el punto A y una fuerza
que forma un ángulo β ¼ 63 ○ con la horizontal se aplica en el punto B.
La magnitud de la fuerza aplicada es P ¼ 80 N. El punto C es el centro
de gravedad de la viga y la viga pesa W ¼ 40 N y tiene una longitud l
¼ 2 m.
Fig. 4.56 Problema
4.10.
CORPORACION UNIVERSITARIA DEL HUILA
CORHUILA

Determine las reacciones generadas en el extremo fijo de la viga.

Respuestas:

Problema 4.11 Considere la viga en forma de L que se ilustra en la


figura 4.57. La viga está soldada a la pared en el punto A, el brazo
AB se extiende en la dirección z positiva y el brazo BC se extiende
en la dirección y negativa. Se aplica una fuerza P en la dirección x
positiva en el extremo libre (punto C) de la viga. Las longitudes de
los brazos AB y BC son ayb, respectivamente, y la magnitud de la
fuerza aplicada es P.
Fig. 4.57 Problema
Suponiendo que el peso de la viga es insignificante, determine las
4.11.
reacciones generadas en el extremo fijo de la viga en términos de a,
by P.

Respuestas: Los componentes de fuerza y momento distintos de cero son:

Problema 4.12 Reconsidere la viga en forma de L que se ilustra en la figura 4.57. Esta vez,
suponga que la fuerza aplicada P tiene componentes en las direcciones x positiva y z positiva
tales que P ¼ Pxi þ Py j. Determine las reacciones generadas en el extremo fijo de la viga en
términos de a, b, Px y Py.

Respuestas:

Problema 4.13 La figura 4.58 ilustra a una persona que


intenta tirar de un bloque en una superficie horizontal con
una cuerda. La cuerda forma un ángulo θ con la horizontal.
Si W es el peso del bloque y μ es el coeficiente de fricción
máxima entre la superficie inferior del bloque y la
Fig. 4.58 Problema
superficie horizontal, demuestre que la magnitud P de la
4.13.
CORPORACION UNIVERSITARIA DEL HUILA
CORHUILA

fuerza mínima que la persona debe aplicar para superar los efectos de fricción y gravitación
(para empezar a mover el bloque) es:

Problema 4.14 Como se muestra en la figura 4.58, considere a una persona que intenta tirar
de un bloque en una superficie horizontal con una cuerda. La cuerda forma un ángulo θ ¼ 15
○ con la horizontal. Si la masa del bloque es m = 50 kg y la fuerza aplicada por el trabajador
es P = 156 N, determine el coeficiente de fricción entre el bloque y la superficie.
Respuesta: μ = 0,34

Problema 4.15 La figura 4.59 ilustra a una persona que intenta


empujar un bloque hacia arriba sobre una superficie inclinada
aplicando una fuerza horizontal. El peso del bloque es W, el
coeficiente de fricción máxima entre el bloque y la pendiente es
μ, y la pendiente forma un ángulo θ con la horizontal.
Determine la magnitud P de la fuerza mínima que debe aplicar
Fig. 4.59 Problema
la persona para superar los efectos de fricción y gravitación (para
4.15.
comenzar a mover el bloque) en términos de W, μ y θ.

Respuesta:

Problema 4.16 Como se muestra en la figura 4.59, considere a una persona que intenta
empujar un bloque hacia arriba sobre una superficie inclinada aplicando una fuerza
horizontal. La pendiente forma un ángulo θ 35 ○ con la horizontal y el coeficiente pf de
fricción entre el bloque y la pendiente es μ = 0,36. Si la fuerza aplicada por la persona para
empujar el bloque hacia arriba por la pendiente es P = 99,8 N,
(a) Determine el peso (W) del bloque.
(b) Determine el cambio en la magnitud de la fuerza aplicada por la persona sobre el
bloque cuando la pendiente forma un ángulo θ = 25° con la horizontal.
CORPORACION UNIVERSITARIA DEL HUILA
CORHUILA

Respuestas: (a) W = 70 N; (b) Disminución del 30,8%


Problema 4.17 Como se muestra en la figura 4.60, una viga
horizontal está articulada a la pared en el punto A. La longitud de
la viga es l = 2 my pesa W 150 N. El punto C es el centro de
gravedad de la viga y es equidistante de ambos extremos. Un cable
se une al grano en el punto B formando un ángulo α 50 ○ con la
horizontal. En el otro extremo, el cable está unido a la pared. Una
carga que pesa W1 50 N se coloca sobre la viga de manera que su
línea de gravedad pase por el punto C. Otra carga del mismo peso
W2 = 50 N se une a la viga en el punto B.
Determine la tensión T en el cable y la fuerza de reacción en el
punto A. Fig. 4.50 Problema
4.17.
Respuesta: T =196 N RA= 161

Problema 4.18 Como se muestra en la figura 4.61, considere una


viga horizontal con bisagras al suelo en el punto A. La longitud
de la viga es l = 4.5 my pesa W = 650 N. El punto C representa el
centro de gravedad del haz y es equidistante del punto A y el
extremo libre del haz (punto B). Se aplica una fuerza P = 850 N
en el punto B formando un ángulo α = 45 ○ con la horizontal.
Además, una carga que pesa W1 125 N se coloca en la viga en su
Fig. 4.51 Problema extremo libre. La distancia entre el punto B y la línea de gravedad
4.18. de la carga es l1 0.3 m. Un rodillo sin fricción se ajusta en el punto
D para restringir la rotación en sentido antihorario de la viga. La
distancia entre los puntos A y D es 12 ¼ 3 m. Calcule las reacciones en la viga en los
puntos A y D.

Respuesta: RA=729 N RD= 239 N

Problema 4.19 Como se muestra en la figura 4.62, considere dos


buzos preparando saltos secuenciales a la piscina. Los buzos están
parados sobre un trampolín horizontal uniforme en los puntos B y C,
respectivamente. El trampolín está sujeto al suelo en el punto A y
sostenido por un rodillo sin fricción en el punto
D. La longitud del trampolín es l ¼ 4 my pesa W ¼ 500 N: La
distancia entre los puntos A y D es l2 1 : 3 m: El centro de gravedad
de la tabla (punto E) es equidistante de los puntos A y
B. El peso de los buzos parados en los puntos B y C es W1 ¼ 680 Fig. 4.52 Problema
4.19.
CORPORACION UNIVERSITARIA DEL HUILA
CORHUILA

N y W2 ¼ 710 N, respectivamente, y la distancia entre los buzos es l1 ¼ 1: 5 m: Determine las


reacciones en el trampolín en los puntos A y D.

Respuesta: RA= 2337 N


RD= 4227 N

Problema 4.20 Como se ilustra en la figura 4.63, considere una viga


horizontal fijada a la pared en el punto A. Se aplica una fuerza F = 135 N
sobre la viga en su extremo libre (punto B), formando un ángulo α =65° con
la horizontal. La longitud de la viga es l =3 my pesa W =150 N. El punto C
es el centro de gravedad de la viga y es equidistante de los puntos A y B.
Dos idénticos cajas que pesan W1 = W2 = 80 N cada una se colocan en la
viga tal que la distancia entre los puntos A y B y las líneas de gravedad de
las cajas l1= l2= 0.75 m: Determine las reacciones en el extremo fijo de la Fig. 4.53 Problema
viga. 4.20.

Respuesta: RA =196 N M =98 Nm

Problema 4.21 Como se ilustra en la figura 4.64, considere un


sencillo dispositivo de tracción aplicado a la pierna de un paciente
de manera que el x el cable de la polea forma un ángulo α = 35° con
la horizontal. La pierna está enyesada y el coeficiente de fricción
entre el yeso y la cama es μ =0.45. El peso de la pierna es W =180 Fig. 4.54 Problema 4.21.
N. Determine la tensión en el cable.
Respuesta: T =75 N

Problema 4.22 Como se ilustra en la figura 4.24, considere un sistema simple de tracción de
tres poleas que se utiliza para transmitir una fuerza horizontal a la pierna de un paciente. La
pierna está enyesada. Un cable se enrolla alrededor de las poleas de modo que un extremo
del cable se une al techo y el otro extremo se une a una bandeja de pesas. El peso del plato de
pesas es W = 12 N. Determine la magnitud de la fuerza horizontal que actúa sobre la pierna.
Respuesta: F = 24
CORPORACION UNIVERSITARIA DEL HUILA
CORHUILA
CORPORACION UNIVERSITARIA DEL HUILA
CORHUILA

Capítulo 5

Aplicaciones de la Estática a la Biomecánica

5.1 Articulaciones Esqueléticas


5.2 Músculos Esqueléticos
5.3 Consideraciones Básicas
5.4 Supuestos Básicos y limitaciones
5.5 Mecánica del codo
5.6 Mecánica del hombro
5.7 Mecánica de la columna v Vertebral
5.8 Mecánica de la cadera
5.9 Mecánica de la rodilla
5.10 Mecánica del tobillo
5.11 Problemas de ejercicio
CORPORACION UNIVERSITARIA DEL HUILA
CORHUILA

5.1 Articulaciones Esqueléticas directo entre ellas y nutrir el


cartílago articular que
El cuerpo humano es rígido en el sentido de que puede recubre las superficies. El
mantener una postura, y flexible en el sentido de que puede cartílago articular, por otro
cambiar de postura y moverse. La flexibilidad del cuerpo lado, es un tejido
humano se debe principalmente a las articulaciones o especializado diseñado para
articulaciones del sistema esquelético. La función principal de aumentar la distribución de
las articulaciones es proporcionar movilidad al sistema la carga en las
musculoesquelético. Además de proporcionar movilidad, una articulaciones y
articulación también debe poseer cierto grado de estabilidad. proporcionar una
Dado que las diferentes articulaciones tienen diferentes superficie
funciones, poseen diversos grados de movilidad y estabilidad.
Algunas articulaciones están construidas para proporcionar
una movilidad óptima. Por ejemplo, la construcción de la
articulación del hombro (rótula) permite que el brazo se
mueva en los tres planos (movimiento triaxial). Sin embargo,
este alto nivel de movilidad se logra a expensas de una
estabilidad reducida, aumentando la vulnerabilidad de la
articulación a lesiones, como dislocaciones. Por otro lado, la
articulación del codo proporciona movimiento principalmente
en un plano (movimiento uniaxial), pero es más estable y
menos propensa a lesiones que la articulación del hombro. El
caso extremo de mayor estabilidad se logra en articulaciones
que no permiten un movimiento relativo entre los huesos que
constituyen la articulación. Las superficies de contacto de los
huesos del cráneo son ejemplos típicos de tales articulaciones.

Las articulaciones del sistema esquelético humano pueden


clasificarse según su estructura y / o función. Las
articulaciones sinartrodiales, como las del cráneo, están
formadas por dos huesos bien ajustados y no permiten ningún
movimiento relativo de los huesos que las forman. Las
articulaciones anfiartrodiales, como las que se encuentran
entre las vértebras, permiten movimientos relativos leves y
presentan una sustancia intermedia (un tejido cartilaginoso o
ligamentoso) cuya presencia elimina el contacto directo de
hueso a hueso. El tercer tipo de articulaciones, el más
importante desde el punto de vista mecánico, se denominan
articulaciones diartrodiales, que permiten diversos grados de
movimiento relativo y tienen cavidades articulares, cápsulas
ligamentosas, membranas sinoviales y líquido sinovial (fig.
5.1). La cavidad articular es el espacio entre los huesos
articulares. La cápsula ligamentosa mantiene unidos los
huesos articulados. La membrana sinovial es el revestimiento
interno de la cápsula ligamentosa que encierra el líquido
sinovial que sirve como lubricante. El líquido sinovial es un
material viscoso que funciona para reducir la fricción, reducir
el desgaste de las superficies articuladas limitando el contacto
CORPORACION UNIVERSITARIA DEL HUILA
CORHUILA

resistente al desgaste que absorbe los golpes.


Varias articulaciones diartrodiales se pueden clasificar
además como deslizantes (por ejemplo, facetas
vertebrales), bisagra (codo y tobillo), pivote
(radiocubital proximal),
condiloide (muñeca),
silla de montar (carpometacarpiano
del pulgar) y rótula
(hombro). y cadera).

Fig. 5.1 Una


articulación
diartrodial: (1) Hueso,
(2) cápsula ligamentosa,
(3, 4) membrana y
líquido sinovial, (5, 6)
cartílago y cavidad
articular
La naturaleza del movimiento alrededor de una articulación diartrodial y la estabilidad
de la articulación dependen de muchos factores, incluida la forma en que las superficies
de articulación encajan, las propiedades de la cápsula articular, la estructura y la longitud
de los ligamentos alrededor de la articulación, y el número y la orientación de los
músculos que cruzan la articulación.
CORPORACION UNIVERSITARIA DEL HUILA
CORHUILA

5.2 Músculos Esqueléticos

En general, hay más de 600 músculos en el cuerpo humano, lo que


representa aproximadamente el 45° del peso corporal total.

Hay tres tipos de músculos: cardíacos, lisos y esqueléticos. El


músculo cardíaco es el tejido contractivo que se encuentra en el
corazón y que bombea la sangre para la circulación. El músculo
liso se encuentra en el estómago, el tracto intestinal y las paredes
de los vasos sanguíneos. El músculo esquelético está conectado a
los huesos del cuerpo y cuando se contrae, hace que los segmentos
del cuerpo se muevan.

El movimiento de los segmentos del cuerpo humano se logra como


resultado de las fuerzas generadas por los músculos esqueléticos
que convierten la energía química en trabajo mecánico. La unidad
estructural del músculo esquelético es la fibra muscular, que está
compuesta por miofibrillas. Las miofibrillas están formadas por
filamentos de actina y miosina. Los músculos exhiben un
comportamiento de material viscoelástico. Es decir, tienen
propiedades materiales tanto sólidas como fluidas. Los músculos
son elásticos en el sentido de que cuando un músculo se estira y
se suelta, recuperará su tamaño y forma originales (sin estirar).
Los músculos son viscosos en el sentido de que existe una
resistencia interna al movimiento.

Un músculo esquelético está unido, a través de tejidos blandos


como aponeurosis y / o tendones, a al menos dos huesos diferentes
que controlan el movimiento relativo de un segmento con respecto
al otro. Cuando sus fibras se contraen bajo la estimulación de un
nervio, el músculo ejerce un efecto de tracción sobre los huesos a
los que está unido. La contracción es una propiedad única del
tejido muscular. En ingeniería mecánica, la contracción implica
acortamiento bajo fuerzas de compresión. En la mecánica
muscular, la contracción puede ocurrir como resultado de un
acortamiento o alargamiento muscular, o puede ocurrir sin ningún
cambio en la longitud del músculo. Además, el resultado de una
contracción muscular es siempre tensión: un músculo solo puede
ejercer un tirón. Los músculos no pueden empujar.

Hay varios tipos de contracciones musculares: una contracción


concéntrica se produce simultáneamente a medida que disminuye
la longitud del músculo (por ejemplo, el bíceps durante la flexión
del antebrazo); se produce una contracción estática mientras la
longitud del músculo permanece constante (el bíceps cuando el
antebrazo se flexiona y se sostiene sin ningún movimiento); y se
produce una contracción excéntrica a medida que aumenta la
longitud del músculo (el bíceps durante la extensión
CORPORACION UNIVERSITARIA DEL HUILA
CORHUILA

del antebrazo). Un músculo puede provocar movimiento solo


mientras su longitud se acorta (contracción concéntrica). Si la
longitud de un músculo aumenta durante una actividad en
particular, entonces la tensión generada por la contracción del
músculo tiene como objetivo controlar el movimiento de los
segmentos corporales asociados con ese músculo
(contracción excéntrica). Si un músculo se contrae, pero no
hay movimiento segmentario, entonces la tensión en el
músculo equilibra los efectos de las fuerzas aplicadas, como
las debidas a la gravedad (contracción isométrica).

Los músculos esqueléticos también se pueden nombrar de


acuerdo con las funciones que cumplen durante una actividad
en particular. Por ejemplo, un músculo se llama agonista si
provoca movimiento a través del proceso de su propia
contracción. Los músculos agonistas son los principales
músculos responsables de generar un movimiento específico.
Un músculo antagonista se opone a la acción de otro músculo.
El músculo sinérgico es el que ayuda al músculo agonista a
realizar el mismo movimiento articular.

5.3 Consideraciones Básicas

En este capítulo, queremos aplicar los principios de la estática


para investigar las fuerzas involucradas en varios grupos de
músculos y articulaciones para diversas posiciones posturales
del cuerpo humano y sus segmentos. Nuestro propósito
inmediato es proporcionar respuestas a preguntas como: ¿qué
tensión deben ejercer los músculos extensores del cuello
sobre la cabeza para sostener la cabeza en una posición
específica? Cuando una persona se dobla, ¿cuál sería la fuerza
ejercida por el erector de la columna sobre la quinta vértebra
lumbar? ¿Cómo varía la compresión en las articulaciones del
codo, la rodilla y el tobillo con las fuerzas aplicadas
externamente y con diferentes disposiciones segmentarias?
¿Cómo varía la fuerza sobre la cabeza femoral con las cargas
que se llevan en la mano? ¿Cuáles son las fuerzas
involucradas en varios grupos de músculos y articulaciones
durante diferentes condiciones de ejercicio?

Las fuerzas involucradas en el cuerpo humano se pueden


agrupar en internas y externas. Las fuerzas internas son las
asociadas con los músculos, ligamentos y tendones, y en las
articulaciones. Las fuerzas aplicadas externamente incluyen el
CORPORACION UNIVERSITARIA DEL HUILA
CORHUILA

efecto de la aceleración gravitacional sobre el cuerpo o sus


segmentos, las fuerzas aplicadas manual y / o mecánicamente
sobre el cuerpo durante el ejercicio y el estiramiento, y las
fuerzas aplicadas al cuerpo mediante prótesis e implementos.
En general, las incógnitas en los problemas estáticos que
involucran el sistema musculoesquelético son las fuerzas de
reacción articular y las tensiones musculares. El análisis
mecánico de una articulación requiere que conozcamos las
características del vector de tensión en el músculo, incluidas
las ubicaciones adecuadas de las inserciones musculares, los
pesos o masas de los segmentos corporales, los centros de
gravedad de los segmentos corporales y el eje anatómico de
rotación del músculo. articulación.
CORPORACION UNIVERSITARIA DEL HUILA
CORHUILA

5.4 Supuestos básicos y limitaciones

El análisis completo de las fuerzas musculares necesarias para mantener diversas


posiciones posturales es difícil debido a la compleja disposición de los músculos dentro
del cuerpo humano y debido a la información limitada. En general, el movimiento relativo
de los segmentos corporales alrededor de una articulación determinada está controlado
por más de un grupo de músculos. Para poder reducir un problema específico de
biomecánica a uno estáticamente determinado y aplicar las ecuaciones de equilibrio, solo
se puede tomar en consideración el grupo de músculos que es la fuente primaria de control
sobre la articulación. Deben ignorarse las posibles contribuciones de otros grupos de
músculos al mecanismo de carga de la articulación. Sin embargo, tenga en cuenta que se
pueden hacer aproximaciones del efecto de otros músculos considerando sus áreas
transversales y sus posiciones relativas en relación con la articulación. Además, si se
conoce la actividad fásica de los músculos a través de algunos experimentos, como las
mediciones de las señales musculares por electromiografía (EMG), se puede estimar la
tensión en diferentes grupos de músculos.

Para aplicar los principios de la estática al análisis de la


mecánica de las articulaciones humanas, adoptaremos los
siguientes supuestos y limitaciones:

• Se conocen los ejes anatómicos de rotación de las


articulaciones.
• Se conocen las ubicaciones de las inserciones musculares.
• Se conoce la línea de acción de la tensión muscular.
• Se conocen los pesos de los segmentos y sus centros de
gravedad.
• Los factores de fricción en las juntas son insignificantes.
• Se ignorarán los aspectos dinámicos de los problemas.
• Solo se considerarán problemas bidimensionales.

Estos análisis requieren que los datos antropométricos sobre


el segmento a analizar estén disponibles. Para este propósito,
existen tablas que enumeran información antropométrica que
incluye pesos, longitudes y centros de gravedad promedio de
los segmentos corporales. Ver Chaffin, Andersson y Martin
(1999) y Winter (2004) para una revisión de los datos
antropométricos disponibles.

De esta discusión se desprende claramente que analizaremos


ciertos problemas idealizados de la biomecánica. Con base en
los resultados obtenidos y la experiencia adquirida, estos
modelos pueden ampliarse teniendo en cuenta factores
adicionales. Sin embargo, un problema dado se volverá más
complejo a medida que se consideren más factores.
CORPORACION UNIVERSITARIA DEL HUILA
CORHUILA

En las siguientes secciones, se aplicarán los principios de la y músculos


estática para analizar las fuerzas involucradas en y alrededor de relacionados, y se
las principales articulaciones del cuerpo humano. Primero, se construirán problemas
proporcionará una breve anatomía funcional de cada articulación biomecánicos
CORPORACION UNIVERSITARIA DEL HUILA
CORHUILA

específicos. Para una discusión más completa sobre la anatomía


funcional de las articulaciones, consulte textos como Nordin y
Frankel (2011) y Thompson (1989). A continuación, se formará una
analogía entre músculos, huesos y articulaciones humanas, y ciertos
elementos mecánicos como cables, vigas y articulaciones mecánicas.
Esto nos permitirá construir un modelo mecánico del sistema
biológico en consideración. Finalmente, el procedimiento descrito en
el Cap. 4.5 se aplicará para analizar el modelo mecánico así
construido. Consulte LeVeau (2010) para ver ejemplos adicionales
de la aplicación de los principios de la estática a la biomecánica.

5.5 Mecánica del Codo

La articulación del codo se compone de tres articulaciones


separadas (Fig. 5.2). La articulación humerocubital es una
articulación en bisagra (ginglymus) formada por la articulación
entre la tróclea en forma de carrete del húmero distal y la fosa
troclear cóncava del cúbito proximal. La estructura de la
articulación humerocubital es tal que solo permite rotaciones
uniaxiales, limitando los movimientos alrededor de la articulación
del codo a flexión (movimiento del antebrazo hacia la parte
superior del brazo) y extensión (movimiento del antebrazo
Fig. 5.2 Huesos del
alejándose de la parte superior del brazo). La articulación
codo: (1) húmero, (2)
humeroradial también es una articulación de bisagra formada entre
capítulo, (3) tróclea,
el capítulo del húmero distal y la cabeza del radio. La articulación
(4) radio, (5) cúbito
radiocubital proximal es una articulación de pivote formada por
la cabeza del radio y la muesca radial del cúbito proximal. Esta
articulación permite que el radio y el cúbito experimenten una
rotación relativa alrededor del eje longitudinal de uno u otro
hueso, dando lugar a la pronación (el movimiento experimentado
al pasar de la palma hacia arriba a la palma hacia abajo) o
supinación (el movimiento experimentado mientras pasando de la
palma hacia abajo a la palma hacia arriba).

Los músculos que coordinan y controlan el movimiento de la


articulación del codo se ilustran en la figura 5.3. El músculo bíceps
braquial es un poderoso flexor de la articulación del codo,
especialmente cuando la articulación del codo está en posición
supina. Es el supinador más poderoso del antebrazo. En el lado Fig. 5.3 Músculos del
distal, el bíceps está unido a la tuberosidad del radio, y en el lado codo: (1) bíceps, (2)
proximal tiene inserciones en la parte superior de la apófisis braquiorradial, (3)
coracoides y el labio superior de la fosa glenoidea. Otro flexor braquial, (4)
importante es el músculo braquial que, independientemente de la pronador redondo, (5)
orientación del antebrazo, tiene la capacidad de producir flexión tríceps braquial, (6)
del codo. Entonces es el flexor más fuerte del codo. Tiene ancóneo,
accesorios en la mitad inferior de la porción anterior del húmero y (7) supinador
la apófisis coronoides del
CORPORACION UNIVERSITARIA DEL HUILA
CORHUILA

braquial. Tiene inserciones en la cabeza inferior de la

Fig. 5.4 Ejemplo 5.1

Fig. 5.4 Fuerzas que


actúan sobre el
antebrazo

cubito. Dado que no se


inserta en el radio, no
puede participar en
pronación o
supinación. El
músculo más
importante que
controla el
movimiento de
extensión del codo es
el músculo tríceps
CORPORACION UNIVERSITARIA DEL HUILA
CORHUILA

cavidad glenoidea del tercio superior del radio.


de la escápula, la
mitad superior de la Las lesiones más comunes del codo incluyen fracturas y
superficie posterior dislocaciones. Las fracturas suelen producirse en los epicóndilos
del húmero, los dos del húmero y en la apófisis olécranon del cúbito. Otro grupo de
tercios inferiores de lesiones del codo se asocia con el uso excesivo, que provoca un
la superficie proceso inflamatorio de los tendones de un codo que ha sido
posterior del dañado por movimientos repetitivos. Estos incluyen los
húmero y la apófisis síndromes del codo de tenista y del codo de golfista.
olécranon del
cúbito. Los
movimientos de
pronación y
supinación del
Ejemplo 5.1 Considere el brazo que se muestra en la figura 5.4.
antebrazo son El codo se flexiona en ángulo recto y se sostiene un objeto en la
realizados por los mano. Las fuerzas que actúan sobre el antebrazo se muestran en
músculos pronador
la figura 5.5a, y el diagrama de cuerpo libre del antebrazo se
redondo y muestra en un modelo mecánico en la figura 5.5b. Este modelo
supinador, asume que el bíceps es el flexor principal y que la línea de acción
respectivamente. El
de la tensión (línea de tracción) en el bíceps es vertical.
pronador redondo
está unido a la parte El punto O designa el eje de rotación de la articulación del codo,
inferior de la cresta que se supone que está fijo a efectos prácticos. El punto A es la
condiloide interna unión del músculo bíceps en el radio, el punto B es el centro de
del húmero, el lado gravedad del antebrazo y el punto C es un punto en el antebrazo
medial del cúbito y que se encuentra a lo largo de una línea vertical que pasa por el
el tercio medio de centro de gravedad del peso en el brazo. mano. Las distancias
la superficie entre el punto O y los puntos A, B y C se miden como a, b y c,
externa del radio. respectivamente. WO es el peso del objeto sostenido en la mano
El músculo
y W es el peso total del antebrazo. FM es la magnitud de la fuerza
supinador tiene
ejercida por el bíceps sobre el radio y FJ es la magnitud de la
inserciones en la
fuerza de reacción en la articulación del codo. Observe que se
cresta condiloide
supone que la línea de acción de la fuerza muscular es vertical.
externa del húmero,
Las fuerzas gravitacionales también son verticales. Por lo tanto,
la parte vecina del
para el equilibrio del antebrazo, la línea de acción de la fuerza de
cúbito y la
reacción articular también debe ser vertical (un sistema de
superficie externa
fuerzas paralelas).

La tarea en este ejemplo es determinar las magnitudes de la tensión


muscular y la fuerza de reacción articular en el codo.

Solución: Tenemos un sistema de fuerzas paralelo y las


incógnitas son las magnitudes FM y FJ de las fuerzas de reacción
CORPORACION UNIVERSITARIA DEL HUILA
CORHUILA

de los músculos y las articulaciones. Considerando el equilibrio


rotacional del antebrazo alrededor de la articulación del codo y
asumiendo que la dirección (cw) es positiva:

∑ 𝑀𝑂 = 0
Es decir, cWO + bW – aFM = 0
Luego FM (Bw + Cwo) (i)
𝑎

Para el equilibrio de traslación del antebrazo en la dirección y:

∑ 𝐹𝑦 = 0 Es decir,
-Fj + FM – W – WO = 0
Luego FJ = FM – W – WO (ii)

Para valores dados de los parámetros geométricos a, b y c, y


pesos W y WO, Ecs. (i) y (ii) se pueden resolver para las
magnitudes de las fuerzas de reacción de los músculos y las
articulaciones. Por ejemplo, suponga que estos parámetros se dan
de la siguiente manera: a = 4 cm, b =15 cm, c = 35 cm, W
=20 N y WO = 80 N.

Luego, de las ecuaciones. (i) y (ii):

Observaciones

• Los resultados numéricos indican que la fuerza ejercida por el


músculo bíceps es aproximadamente diez veces mayor que el
peso del objeto sostenido en la posición considerada. En relación
con el eje de la articulación del codo, la longitud a del brazo de
palanca de la fuerza muscular es mucho menor que la longitud c
del brazo de palanca disfrutado por la carga sostenida en la mano.
Cuanto más pequeño sea el brazo de palanca, mayor será la
tensión muscular necesaria para equilibrar el efecto de rotación
en el sentido de las agujas del reloj de la carga sobre la
articulación del codo. Por lo tanto, durante el levantamiento, es
desventajoso tener un accesorio muscular cerca de la articulación
del codo. Sin embargo, cuanto más cerca esté el músculo de la
articulación, mayor será el rango de movimiento de flexión-
extensión del codo y más rápido el extremo distal (mano) del
antebrazo puede alcanzar su objetivo de moverse hacia la parte
superior del brazo o el hombro.
CORPORACION UNIVERSITARIA DEL HUILA
CORHUILA

Fig. 5.6
Componentes
rotacionales (FMn)
y estabilizadores o
deslizantes (FMt) de
la fuerza muscular
CORPORACION UNIVERSITARIA DEL HUILA
CORHUILA

Fig. 5.7 Explicando rotación. Suponga que el sistema de articulación que se muestra en
la fuerza de reacción la figura 5.6a ilustra la posición del antebrazo en relación con la
articular en el codo. parte superior del brazo. n designa una dirección perpendicular
(normal) al eje largo del antebrazo y t es tangente a él. Suponiendo
que la línea de acción de la fuerza muscular permanece paralela al
eje mayor del húmero, la FM se puede descomponer en sus
componentes rectangulares F-Mn y FMt. En este caso, F-Mn es el
El ángulo entre la componente de rotación (rotatorio) de la fuerza muscular porque su
línea de acción de función principal es rotar el antebrazo alrededor de la articulación
la fuerza muscular del codo. El componente tangencial F-Mt de la fuerza muscular actúa
y el eje para comprimir la articulación del codo y se denomina componente
longitudinal del estabilizador de la fuerza muscular. A medida que el ángulo de
hueso sobre el que tracción se acerca a 90, la magnitud del componente rotacional de
se ejerce la fuerza la fuerza muscular aumenta mientras que su componente
muscular se estabilizador disminuye, y cada vez se “desperdicia” menos energía
denomina ángulo para comprimir la articulación del codo. Como se ilustra en la figura
de tracción y es 5.6b, la función estabilizadora de F-Mt cambia a una función de
fundamental para deslizamiento o dislocación cuando el ángulo entre los ejes largos
determinar la del antebrazo y la parte superior del brazo es menor de 90°.
eficacia de la
fuerza muscular.
Cuando la parte
inferior del brazo • El codo es una articulación diartrodial (sinovial). Una
se flexiona en cápsula ligamentosa encierra una cavidad articular que está llena
ángulo recto, la de líquido sinovial. El líquido sinovial es un material viscoso cuya
tensión muscular función principal es lubricar las superficies de articulación,
solo tiene un efecto reduciendo así las fuerzas de fricción que pueden desarrollarse
de rotación en el mientras una superficie de articulación se desliza sobre la otra. El
antebrazo líquido sinovial también nutre los cartílagos articulares. Una
alrededor de la
articulación del
codo, porque la
línea de acción de
la fuerza muscular
forma un ángulo
recto con el eje
longitudinal del
antebrazo. Para
otras posiciones
flexionadas del
antebrazo, la
fuerza muscular
puede tener un
componente de
traslación
(estabilizador o
deslizante) así
como un
componente de
CORPORACION UNIVERSITARIA DEL HUILA
CORHUILA

propiedad común de Por tanto, la fuerza de reacción conjunta FJ corresponde a la


los fluidos es que resultante del sistema de fuerzas distribuidas (presión)
ejercen presiones aplicada a
(fuerza por unidad
de área) que se
distribuyen sobre las
superficies que
tocan. La presión del
fluido siempre actúa
en una dirección
hacia y
perpendicular a la
superficie que toca
teniendo un efecto
de compresión sobre
la superficie. Note
que en la figura 5.7,
los pequeños
vectores que indican
la presión del fluido
tienen componentes
en las direcciones
horizontal y vertical.
Determinamos que
la fuerza de reacción
de la articulación en
el codo actúa
verticalmente hacia
abajo sobre el
cúbito. Esto implica
que las componentes
horizontales de estos
vectores se cancelan
(es decir, la mitad
apuntando a la
izquierda y la otra
mitad apuntando a la
derecha), pero sus
componentes
verticales (en el
cúbito, casi todos
apuntan hacia abajo)
se suman para
formar la resultante
fuerza FJ (que se
muestra con una
flecha discontinua
en la figura 5.7c).
CORPORACION UNIVERSITARIA DEL HUILA
CORHUILA

través del líquido braquial y braquiorradial con inserciones en los puntos A1, A2 y A3,
sinovial. respectivamente. Sean θ1, θ2 y θ3 los ángulos que forman los
• La músculos bíceps, braquial y braquiorradial con el eje largo del
simplificación antebrazo. En comparación con el sistema de un solo músculo que
más crítica constaba de dos incógnitas (FM y FJ), el análisis de este sistema de
realizada en este tres músculos es bastante complejo. En primer lugar, este no es un
ejemplo es que se simple sistema de fuerzas paralelas. Incluso si asumimos que las
asumió que el ubicaciones de las inserciones musculares (A1, A2 y A3), sus
bíceps era el ángulos de tracción (θ1, θ2 y θ3) y las longitudes de sus brazos de
único grupo momento (a1, a2 y a3) medidos desde el Si se conoce la articulación
muscular del codo, todavía hay cinco incógnitas en el problema (FM1, FM2 y
responsable de Fj y β, donde el ángulo β es un ángulo entre FJ y los ejes largos del
mantener la antebrazo). El número total de ecuaciones disponibles de estática
configuración es tres:
flexionada del
antebrazo. La
razón para hacer ∑ 𝑀𝑂 = 0: a1 FM1 = a2FM2 + a3FM3 = Bw + Cwo (iii)
tal suposición fue ∑ 𝐹𝑥 = 0: Fjx = FM1x + FM2x + FM3x (iv)
reducir el sistema ∑ 𝐹𝑦 = 0: Fjy = FM1y + FM2y + FM3y – W – WO (iv)
en consideración
a uno que está Tenga en cuenta que una vez que se determinan las fuerzas
estáticamente musculares, las ecuaciones (iv) y (v) producirán los componentes
determinado. En de la fuerza de reacción conjunta F-j. En lo que respecta a las
realidad, además fuerzas musculares, solo tenemos la ecuación (iii) con tres
del bíceps, el incógnitas. En otras palabras, tenemos un problema estáticamente
braquial y el indeterminado. Para obtener una solución única, necesitamos
braquiorradial información adicional relacionada con FM1, FM2 y FM3.
son músculos
flexores Puede haber varios enfoques para la solución de este problema.
primarios del Los criterios para estimar la distribución de fuerza entre diferentes
codo. grupos de músculos pueden establecerse mediante: (1) el uso de
áreas transversales de los músculos, (2) el uso de mediciones de
Considere la electromiografía (EMG) de las señales musculares y (3) la
posición aplicación de ciertas técnicas de optimización. Se puede suponer
flexionada del que cada músculo ejerce una fuerza
brazo que se
muestra en la
figura 5.8a. El
diagrama de
cuerpo libre del
antebrazo se
muestra en la
figura 5.8b. FM1,
FM2 y FM3 son las
magnitudes de las
fuerzas ejercidas
sobre el
antebrazo por los
músculos bíceps,
CORPORACION UNIVERSITARIA DEL HUILA
CORHUILA

Fig. 5.8 Sistema de


tres músculos

Fig. 5.9 El hombro: (1) articulación


esternoclavicular, (2)
esternón, (3)
articulación

glenohumeral, (4)
clavícula, (5)
articulación

acromioclavicular, (6)
proceso del acromion, (7) fosa glenoidea, (8) escápula, (9)
húmero FM3 = k31 FM1 con k31 = 𝑆3 (vii)
proporcional a su área de sección transversal. Si S1, S2 y S3 son
las áreas transversales del bíceps, el braquial y el braquiorradial,
entonces este criterio puede aplicarse expresando las fuerzas
musculares de la siguiente manera:
CORPORACION UNIVERSITARIA DEL HUILA
CORHUILA

FM2 = k21 FM1 con k21 = 𝑆2 (vi)


𝑆1

𝑆1
CORPORACION UNIVERSITARIA DEL HUILA
CORHUILA

minimizar las fuerzas ejercidas, los momentos sobre las


Si se conocen las articulaciones (para situaciones dinámicas) y / o el trabajo. hecho
constantes k21 y k31, por los músculos. La pregunta es, ¿qué distribución de fuerzas entre
entonces las los distintos músculos facilita la máxima eficiencia? Estos
ecuaciones. (vi) y conceptos y referencias relevantes se discutirán brevemente en la
(vii) se pueden secc.
sustituir en ecuación 5.11.
(iii), que luego se
puede resolver para
FM1: 5.6 Mecánica del hombre

F La estructura ósea y los músculos del complejo del hombro se


M1 = ilustran en las Figs. 5.9 y 5.10. El hombro forma la base de todos
𝑑𝑊+𝑐𝑊 los movimientos de las extremidades superiores. La compleja
𝑂 𝑎1+ estructura del hombro se puede dividir en dos: la articulación del
𝑎2 hombro y la cintura escapular.
𝑘21+
𝑎3 La articulación del hombro, también conocida como articulación
𝑘31 glenohumeral, es una articulación esférica entre la cabeza (bola)
humeral casi hemisférica y la fosa glenoidea (cavidad) poco
propósito es realizar una determinada tarea (estática o dinámica) de la
Sustituyendo FM1 manera más eficiente, entonces los músculos del cuerpo deben actuar
nuevamente en las para
ecuaciones (vi) y (vii)
producirán entonces
las magnitudes de las
fuerzas en los
músculos braquial y
braquiorradial. Los
valores de k21 y k31
también se pueden
estimar usando las
amplitudes de las
señales EMG del
músculo.

Este problema
estáticamente
indeterminado
también puede
resolverse
considerando algunas
técnicas de
optimización. Si el
CORPORACION UNIVERSITARIA DEL HUILA
CORHUILA

cóncava de la hace que la articulación sea más susceptible a inestabilidad y


escápula. La poca lesiones, como dislocaciones. La estabilidad de la articulación es
profundidad de la proporcionada por los ligamentos glenohumeral y coracohumeral,
fosa glenoidea y por los
permite una gran
libertad de
movimiento de la
cabeza humeral en
la superficie músculos que cruzan
articulada de la la articulación. El mayor
glenoides. Los
movimientos Los músculos de la articulación del hombro son: deltoides,
permitidos son: en supraespinoso, pectoral mayor, coracobraquial, latissimus dorsi,
el plano a sagital, redondo mayor, redondo menor, infraespinoso y subescapular.
flexión
(movimiento del La estructura ósea de la cintura escapular consta de la clavícula
húmero hacia el (clavícula) y la escápula (omóplato). La articulación
frente, un acromioclavicular es una pequeña articulación sinovial entre la
movimiento hacia clavícula distal y el proceso acromio de la escápula. La estabilidad
adelante hacia de esta articulación se ve reforzada por los ligamentos
arriba) y extensión coracoclaviculares. La articulación esternoclavicular es la
(retorno de la articulación entre el manubrio del esternón y la clavícula proximal.
flexión); en el La estabilidad de esta articulación se ve reforzada por el ligamento
plano coronal,
abducción
(movimiento
horizontal hacia
arriba del húmero
para el lado) y
aducción (regreso
de la abducción); y
en el plano
transversal,
rotación hacia
afuera
(movimiento del
húmero alrededor
de su eje
longitudinal hacia
el lado lateral) y
rotación hacia
adentro (retorno de
la rotación hacia
afuera). La
configuración de
las superficies de
articulación de la
articulación del
hombro también
CORPORACION UNIVERSITARIA DEL HUILA
CORHUILA

costoclavicular. La coordinan estos movimientos son el trapecio, el elevador de la


articulación escápula, el romboide, el pectoral menor, el
acromioclavicular y
la articulación
esternoclavicular
tienen capas de
cartílago, llamadas
meniscos,
interpuestas entre
sus superficies
óseas.

Hay cuatro pares de


movimientos
escapulares:
elevación
(movimiento de la
escápula en el plano
frontal) y depresión
(retorno desde la
elevación), rotación
hacia arriba (girando
la fosa glenoidea
hacia arriba y el
borde medial
inferior de la
escápula alejándolo
de la columna
vertebral) y rotación
hacia abajo (regreso
de la rotación hacia
arriba),
prolongación
(movimiento del
extremo distal de la
clavícula hacia
adelante) y
retracción (regreso
de la prolongación),
y rotación hacia
adelante y hacia
atrás (rotación de la
escápula alrededor
del eje de la
clavícula). Algunos
de los principales
músculos que
controlan y
CORPORACION UNIVERSITARIA DEL HUILA
CORHUILA

serrato anterior y el deltoides está unido al húmero, el punto B es el centro de gravedad


subclavio. de todo el brazo y el punto C es el centro de gravedad de la
mancuerna. W es el peso del brazo, WO es el peso de la mancuerna,
FM es la magnitud de la tensión en el músculo deltoides y FJ es la
magnitud de la fuerza de reacción articular en el hombro. La
Ejemplo 5.2 resultante de la fuerza del músculo deltoides forma un ángulo θ
Considere a una con la horizontal. Las distancias entre el punto O y los puntos A,
persona que B y C se miden como a, b y c, respectivamente.
fortalece los
músculos del Determine la magnitud FM de la fuerza ejercida por el músculo
hombro deltoides para mantener el brazo en la posición que se muestra.
mediante También determina
ejercicios con
mancuernas. La
figura 5.11
ilustra la
posición del
brazo izquierdo
cuando el brazo
está
completamente
abducido a la
horizontal. El
diagrama de
cuerpo libre del
brazo se muestra
en la figura 5.12
junto con un
modelo
mecánico del
brazo. También
en la figura 5.12,
las fuerzas que
actúan sobre el
brazo se
descomponen en
sus componentes Fig. 5.10 Músculos del hombro: (1) deltoides, (2) pectoral menor,
rectangulares a (3) subescapular, (4) pectoral mayor, (5) trapecio, (6)
lo largo de las infraespinoso y redondo menor, (7) latissimus dorsi, (8) elevador
direcciones de la escápula, (9) supraespinoso, (10) romboideo, (11) redondo
horizontal y mayor
vertical. El punto
O corresponde al
eje de rotación
de la articulación
del hombro, el
punto A es
donde el
músculo
CORPORACION UNIVERSITARIA DEL HUILA
CORHUILA

Fig. 5.12 Fuerzas que actúan sobre el brazo y un modelo


mecánico que
representa el brazo

la magnitud y dirección de la fuerza de reacción en la articulación


del hombro en términos de parámetros especificados.

Fig. 5.11 El brazo


está en Solución: Con respecto al marco de coordenadas xy, las fuerzas de
abducción horizontal reacción del músculo y la articulación tienen dos componentes,
mientras que los pesos del brazo y la mancuerna actúan en la
dirección y negativa. Los componentes de la fuerza muscular son:

FMx = FM cos θ (-x) (i)


FMy = FM sin θ (+y) (ii)

Los componentes de la fuerza de reacción conjunta son:

β es el ángulo que forma la fuerza de reacción conjunta con la


horizontal. Se conocen la línea de acción y la dirección (en términos
de θ) de la fuerza ejercida por el músculo en el brazo. Sin embargo,
la magnitud FM de la fuerza muscular, la magnitud FJ y la dirección
(β) de la fuerza de reacción articular son desconocidas. Tenemos un
total de tres incógnitas, FM, FJ y β (o FM, FJx y FJy). Para poder
resolver este problema bidimensional, tenemos que utilizar las tres
ecuaciones de equilibrio.

Primero, considere el equilibrio rotacional del brazo


alrededor de la articulación del hombro en el punto O. La
fuerza de reacción de la articulación no produce torque
alrededor del punto O porque su línea de acción pasa por el
punto O. Para propósitos prácticos, podemos despreciar la
posible contribución de la horizontal componente de la
fuerza muscular al momento generado alrededor del punto
O suponiendo que su línea de acción también pasa por el
punto O. Nótese que este no es un supuesto crítico o
necesario para resolver este problema. Si supiéramos la
longitud de su brazo de momento (es decir, la distancia
vertical entre O y A), podríamos incorporar fácilmente el
par generado por FMy sobre el punto O en el análisis. Bajo
estas consideraciones, solo hay tres fuerzas que producen
CORPORACION UNIVERSITARIA DEL HUILA
CORHUILA

momentos
alrededor del
punto O. Para
el equilibrio
rotacional del
brazo, el
momento
neto
alrededor del
punto O debe
ser igual a
cero.
Tomando los
momentos en
sentido
antihorario
para ser
positivos:
CORPORACION UNIVERSITARIA DEL HUILA
CORHUILA

∑ 𝑀𝑂 = 0 aFMy – bW – cWO = 0
(bW + cWO) (v) a

Para dados a, b, c, W y WO, ecuación (v) se puede utilizar


para determinar el componente vertical de la fuerza ejercida
por el músculo deltoides. La ecuación (ii) ahora se puede
utilizar para determinar la fuerza total ejercida por el
músculo:

FM = 𝐹𝑀𝑦 (vi)
𝑆𝑖𝑛 𝜃
Conociendo FM, ecuación (i) cederá el componente
horizontal de la tensión en el músculo:

FMx = FM cos θ (vii)

Los componentes de la fuerza de reacción conjunta se pueden


determinar considerando el equilibrio de traslación del brazo en
las direcciones horizontal y vertical:

∑ 𝐹𝑥 = 0 es decir: FJx – FMx = 0 luego FJx = FMx (vii)


∑ 𝐹𝑦 = 0 es decir: - FJy + FMy – W – WO = 0 Luego
FJy = FMy – W – WO

Conocer los componentes rectangulares de la fuerza de reacción


conjunta nos permite calcular la magnitud de la fuerza en sí y el
ángulo que forma su línea de acción con la horizontal:

FJ = √(𝐹𝐽𝑥)2 + (𝐹𝐽𝑦)2 (x)

𝐹
(xi)

𝐹𝐽𝑥
CORPORACION UNIVERSITARIA DEL HUILA
CORHUILA

Ahora considere que a = 15 cm, b = 30 cm, c = 60 cm, θ = 15°, W =


40N y WO = 60 N. Entonces:
CORPORACION UNIVERSITARIA DEL HUILA
CORHUILA

FMy
= 320N (+y)
FM = 1236N
𝑆𝑖𝑛 15°
FMx (1236) (cos 15°) = 1194N (-x)
FJx = (1194N (+x)
FJy = 320 – 40 – 60 = 220 N (-y)
FJ =
1214N ) = 10°

Observaciones

• FMx es el componente estabilizador y FMy es el componente


rotacional del músculo deltoides. FMx es aproximadamente cuatro
veces más grande que FMy. Un gran componente estabilizador
sugiere que la posición horizontal del brazo no es estable y que el
músculo necesita ejercer una gran fuerza horizontal para
estabilizarlo.
CORPORACION UNIVERSITARIA DEL HUILA
CORHUILA
CORPORACION UNIVERSITARIA DEL HUILA
CORHUILA

Fig. 5.13 La columna lesiones. El húmero es particularmente vulnerable a las


vertebral: (1) lesiones debido a su configuración desprotegida.
vértebras cervicales,
(2) vértebras
torácicas, (3)
vértebras lumbares,
(4) sacro

• El hombro
humano es muy
susceptible a lesiones.
Las lesiones más
frecuentes son las
dislocaciones de la
articulación del
hombro y la fractura
del húmero. Dado que
la cavidad de la
articulación
glenohumeral es poco
profunda, la cabeza del
húmero tiene relativa
libertad para rotar
alrededor de la
superficie de
articulación de la fosa
glenoidea. Esta
libertad de movimiento
se consigue, sin
embargo, mediante la
reducción de la
estabilidad articular.
La cabeza humeral
puede desplazarse de
diversas formas, según
la fuerza o debilidad de
la estructura muscular
y ligamentosa del
hombro y según la
actividad física. Las
fracturas humerales

son otro tipo común de


CORPORACION UNIVERSITARIA DEL HUILA
CORHUILA

• Los rangos lumbar de la columna vertebral forman el tronco. Las regiones


de movimiento sacra y coccígea están unidas con la pelvis y pueden considerarse
promedio del brazo partes de la cintura pélvica.
alrededor de la
articulación del La columna vertebral consta de 24 vértebras intrincadas y
hombro son 230° complejas (fig. 5.13). Las articulaciones entre las vértebras son
durante la flexión- articulaciones anfiartrodiales. Se interpone un disco
extensión y 170° fibrocartilaginoso entre cada par de vértebras. Las funciones
tanto en abducción- principales de estos discos intervertebrales son soportar las cargas
aducción como en transmitidas desde los segmentos superiores, actuar como
rotación hacia amortiguadores, eliminar el contacto de hueso con hueso y reducir
adentro-hacia los efectos de las fuerzas de impacto al evitar el contacto directo
afuera. entre las estructuras óseas de las vértebras. Las articulaciones de
cada vértebra con las vértebras adyacentes permiten el
5.7 Mecánica de la movimiento en tres planos, y toda la columna funciona como una
columna sola articulación de rótula. La estructura de la columna permite
vertebral una amplia variedad de movimientos que incluyen flexión-
extensión, flexión lateral y rotación.
La columna
vertebral humana es Dos articulaciones particularmente importantes de la columna
la parte más vertebral son las que tienen la cabeza (hueso occipucio del cráneo)
compleja del y la primera vértebra cervical, atlas, y el atlas y la segunda
sistema vértebra, el eje. La articulación atlantooccipital es la unión entre
musculoesquelético la primera vértebra cervical (el atlas) y el hueso occipital de la
humano. Las cabeza. Se trata de una articulación condiloide doble y permite los
principales movimientos de la cabeza en los planos sagital y frontal. La
funciones de la articulación atlantoaxial es la unión entre el atlas y la apófisis
columna vertebral odontoides de la cabeza. Es una articulación de pivote, que
son proteger la permite a la cabeza
médula espinal;
para sostener la
cabeza, el cuello y
las extremidades
superiores; los planos sagital y frontal. La articulación atlantoaxial es la
transferir cargas de unión entre el atlas y la apófisis odontoides de la cabeza. Es una
la cabeza y el tronco articulación de pivote, que permite que la cabeza gire en el plano
a la pelvis; y transversal. Los grupos de músculos que proporcionan, controlan
permitir una y coordinan el movimiento de la cabeza y el cuello son los
variedad de prevertebrales (anterior), hioides (anterior),
movimientos. La esternocleidomastoideo (anterior-lateral), escaleno (lateral),
columna vertebral elevador de la escápula (lateral), suboccipital (posterior). y spleni
consta de las (posterior).
regiones cervical
(cuello), torácica
(pecho), lumbar
(parte inferior de la
espalda), sacra y
coccígea. Las
secciones torácica y
CORPORACION UNIVERSITARIA DEL HUILA
CORHUILA

la columna lumbar. La flexión lateral del tronco resulta de las


La columna obtiene acciones de los músculos abdominales y posteriores. El
estabilidad de los movimiento de rotación del tronco es controlado por la acción
discos simultánea de los músculos anterior y posterior.
intervertebrales y de La columna vertebral es vulnerable a diversas lesiones. La lesión
los ligamentos y más grave afecta a la médula espinal, que está sumergida en
músculos líquido y protegida por la estructura ósea. Otras lesiones críticas
circundantes (fig. incluyen vértebras fracturadas y discos intervertebrales
5.14). Los discos y herniados. El dolor lumbar también puede ser el resultado de
ligamentos tensiones en las regiones inferiores de la columna.
proporcionan

estabilidad intrínseca
y los músculos
proporcionan soporte también apoyan la columna contra los efectos
extrínseco. Los de la gravedad. El músculo cuadrado lumbar es
músculos de la importante en la flexión lateral del tronco.
columna existen en También estabiliza la pelvis y
pares. La porción
anterior de la columna
contiene los músculos
abdominales: el recto
del abdomen, el
transverso del
abdomen, los
oblicuos externos y
los oblicuos internos.
Estos músculos
proporcionan la
fuerza necesaria para
la flexión del tronco y
mantienen los
órganos internos en la
posición adecuada.
Hay tres capas de
músculos posteriores
del tronco: erector de
la columna,
semiespinoso y
grupos de músculos
espinales posteriores
profundos. La función
principal de los
músculos ubicados en
la parte posterior de la
columna es
proporcionar
extensión del tronco.
Estos músculos
CORPORACION UNIVERSITARIA DEL HUILA
CORHUILA

Ejemplo 5.3 ¿Cuál es la fuerza de compresión aplicada sobre la primera


Considere la vértebra cervical en la articulación atlantooccipital?
posición de la
cabeza y el cuello
que se muestran en
la figura 5.15.
También se
muestran las
fuerzas que actúan
sobre la cabeza. La
cabeza pesa W =
50N y su centro de
gravedad se
encuentra en el
punto C. FM es la
magnitud de la
fuerza resultante
ejercida por los
músculos
extensores del
cuello, que se
aplica sobre el
cráneo en el punto
A. El centro de la
articulación
atlantooccipital
está ubicado en el
punto B. Para esta
posición
flexionada de la
cabeza, se estima
que la línea de
acción de la fuerza
del músculo del
cuello forma un
ángulo θ = 30° y la
línea de acción de
la fuerza de
reacción articular
forma un ángulo β
= 60° con la
horizontal.

¿Qué tensión
deben ejercer los
músculos
extensores del
cuello para
sostener la cabeza?
CORPORACION UNIVERSITARIA DEL HUILA
CORHUILA

Fig. 5.14 Músculos


seleccionados del
cuello y la columna fuerzas que actúan sobre la cabeza
vertebral:
(1) esplenio, (2)
esternocleidomastoide
o, (3) hioides, (4)
elevador de la
escápula, (5) erector
de la columna, (6)
oblicuos, (7) recto del
abdomen, (8)
transverso
del abdomen

Fig. 5.15 Las fuerzas


sobre el cráneo forman
un sistema concurrente

Fig. 5.16
Componentes de las
CORPORACION UNIVERSITARIA DEL HUILA
CORHUILA

Solución: Tenemos que es también elegido para ser el origen del marco de coordenadas
un sistema de tres xy. Los componentes rectangulares de las fuerzas de reacción del
fuerzas con dos músculo y la articulación en las direcciones x y y son:
incógnitas:
magnitudes FM y FJ de FMx = FM cos θ (i)
las fuerzas de FMy = FM sin β (ii)
reacción de los FJx = FJ cos θ (iii)
músculos y las
articulaciones. Dado
que el problema tiene
una geometría
relativamente
complicada, es
conveniente utilizar
la condición de que
para que un cuerpo
esté en equilibrio, el
sistema de fuerzas
que actúa sobre él
debe ser concurrente
o paralelo. En este
caso, está claro que
las fuerzas
involucradas no
forman un sistema de
fuerzas paralelo. Por
tanto, el sistema de
fuerzas considerado
debe ser concurrente.
Recuerde que un
sistema de fuerzas es
concurrente si las
líneas de acción de
todas las fuerzas
tienen un punto
común de
intersección.

En la figura 5.15, las


líneas de acción de las
tres fuerzas que
actúan sobre la
cabeza se extienden
para encontrarse en el
punto O. En la figura
5.16, las fuerzas W,
FM y FJ que actúan
sobre el cráneo se
trasladan al punto O,
CORPORACION UNIVERSITARIA DEL HUILA
CORHUILA

Las condiciones de equilibrio de traslación en las direcciones x


y y producirán:

∑ 𝐹𝑥 = 0 : es decir: - FJx + FMx = 0, luego FJx = FMx (v)


∑ 𝐹𝑦 = 0 : es decir: - W- FMy + FJy = 0, luego FJy = W + FMy
(vi)

Sustituir ecuación (i) y (iii) en la ecuación. (v):

FJ = cos β = FM cos θ (vii)

Sustituir ecuación (ii) y (iv) en ecuación (vi):

FJ = sin β = W +FM sin θ (viii)

Sustituya esta ecuación en la ecuación (viii), es decir:

𝐹𝑀 . 𝑐𝑜𝑠 𝜃
. sin β = W + FM sin θ,
𝑐𝑜𝑠 𝛽
FM . cos θ tan β = W + FM sin θ, luego
tan β = 𝑊+ 𝐹𝑀 𝑠𝑖𝑛 𝜃 = (ix)
𝐹𝑀 𝑐𝑜𝑠 𝜃

Equation (ix) can now be solved for the unknown muscle force
FM:

FM . cos θ tan β = W + FM sin θ

FJy = FJ sin β
(iv)
CORPORACION UNIVERSITARIA DEL HUILA
CORHUILA

FM (cos θ tan β - ecuación (iii) o ecuación (iv). Usando la ecuación (iii):


sin θ) = W

FM = y (vi):

FJx = 43 N (-N) FJy = 50 + 25 =


𝑊 75 N (+y)
(x)
𝑐𝑜 La resultante de la fuerza de reacción conjunta se puede calcular
𝑠 a partir de la
𝜃
𝑡𝑎
𝑛
𝛽

𝑠𝑖
𝑛
𝜃

La ecuación (x) da la
tensión en el músculo
en función del peso W
de la cabeza y los
ángulos θ y β que las
líneas de acción del
músculo y las fuerzas
de reacción
articulares forman
con la horizontal. La
sustitución de los
valores numéricos de
W, θ y β producirá:

FM =

= 50 N

De las ecuaciones (i) y


(ii):

FMx = (50) ( cos


30°) = 43 N (+x)
FMy = (50) (sin
30°) = 25 N (-y)

De las ecuaciones (v)


CORPORACION UNIVERSITARIA DEL HUILA
CORHUILA

F se muestra en la figura 5.17. FJn es la magnitud de la componente


J normal de FJ que comprime la superficie de la articulación, y FJt
= es la magnitud de su componente tangencial que tiene un efecto
cortante sobre las superficies de la articulación. Las fuerzas en
los músculos y ligamentos del cuello actúan para contrarrestar
𝐹 este efecto de cizallamiento.
𝐽
𝑥
=
4
3 Ejemplo 5.4 Considere el levantador de pesas ilustrado en la
= figura 5.18, que está inclinado hacia adelante y levanta un peso
8 W0. En la posición que se muestra, el tronco del atleta se flexiona
6 en un ángulo θ medido desde la posición erguida
N (vertical).
c
o
s
𝛽
c
o
s
6
0
°

Observaciones

• Los músculos
extensores de la
cabeza deben aplicar
una fuerza de 50 N
para sostener la
cabeza en la posición
considerada. La
fuerza de reacción
desarrollada en la
articulación
atlantooccipital es de
aproximadamente 86
N.

• La fuerza de
reacción de la
articulación se puede
descomponer en dos
componentes
rectangulares, como
CORPORACION UNIVERSITARIA DEL HUILA
CORHUILA

Fig. 5.17 Componentes


normal y cortante de la Fig. 5.18 Un levantador de pesas
fuerza de reacción
conjunta

Fig.5.20 Diagrama de cuerpo libre

Fig. 5.19 Fuerzas que


actúan sobre la parte
inferior del cuerpo del
atleta
Las fuerzas que actúan sobre la parte inferior del cuerpo del atleta
CORPORACION UNIVERSITARIA DEL HUILA
CORHUILA

se muestran en la
figura 5.19
considerando una
sección que pasa por
la quinta vértebra
lumbar. En la figura
5.20 se ilustra un
modelo mecánico de
la parte inferior del
cuerpo del atleta (la
pelvis y las piernas)
junto con los
parámetros
geométricos del
problema en
consideración. W es
el peso total del
atleta, W1 es el peso
de las piernas
incluida la pelvis, (W
+ W0) es la fuerza de
reacción total del
suelo aplicada al
atleta a través de los
pies (en el punto C),
FM es la magnitud de
la fuerza resultante
ejercida por los
músculos erectores
de la columna que
sostienen el tronco, y
FJ es la magnitud de
la fuerza de
compresión
generada en la unión
(punto O) del sacro y
la quinta vértebra
lumbar. El centro
CORPORACION UNIVERSITARIA DEL HUILA
CORHUILA

de gravedad de las determinar la magnitud FM de la fuerza resultante ejercida por los


piernas, incluida la músculos erectores de la columna. Considerando que los momentos
pelvis, se encuentra en el en el sentido
punto B. En relación con
el punto O, las
longitudes de los brazos
de palanca de la fuerza
muscular, el peso
inferior del cuerpo y la
fuerza de reacción del
suelo se miden como a, b
y c respectivamente.

Suponiendo que la línea


de tracción de la fuerza
muscular resultante
ejercida por los
músculos erectores de la
columna es paralela al
tronco (es decir,

formando un ángulo θ
con la vertical),
determine FM y FJ en
términos de b, c, θ, W0,
W1, y W.

Solución: En este caso,


hay tres incógnitas: FM,
FJx y FJy. Las
longitudes de los brazos
de palanca de la fuerza
muscular, la fuerza de
reacción del suelo y la
fuerza gravitacional de
las piernas, incluida la
pelvis, se dan como
medidas desde el punto
O. Por lo tanto, podemos
aplicar la condición de
equilibrio rotacional
sobre el punto O para
CORPORACION UNIVERSITARIA DEL HUILA
CORHUILA

de las agujas del reloj determinar utilizando las condiciones de equilibrio de traslación
son positivos: de la parte inferior del cuerpo del atleta en las direcciones x y y:

∑ 𝑀𝑂 = 0: aFM + bW1 -
c (W + W0) = 0 ∑ 𝐹𝑦 = 0: Es decir, FMy - FJy - W1 + (W + WO)= 0,
luego FJy = FMy + W + WO – W1 (iv)
Resolver esta ecuación
para FM producirá:

FM = 𝑐 Suponga que en un instante el atleta se dobla de modo que su


(𝑊+ 𝑊0 )𝑏𝑊1 (i) tronco forme un ángulo θ = 45° con la vertical, y que las
𝑎 longitudes de los brazos de palanca se miden en términos de la
altura h del atleta y los pesos se dan en términos del peso W del
atleta como: a = 0.02h, b = 0.08h, c= 0.12h, W0 = W; y W1 =
Para valores numéricos 0.4W:
dados de b, c, θ, W0, W1, Usando la ecuación (i):
y W, ecuación
(i) se puede utilizar
para determinar la FM = (1.12ℎ)(𝑊+
magnitud de la fuerza 𝑊)−(0.08ℎ)(0.4𝑊) =
muscular resultante.
Una vez que se calcula 10.4 W 0.02ℎ
FM, sus componentes
en las direcciones x y
De las ecuaciones (ii) y (iii):
y se pueden
determinar usando:
FMx = (10.4W) (sin 45°) = 7.4W
FMx = FM sin θ FMy = (10.4W) (cos 45°) =
(i 7.4W
i)
FMy = FM cos θ
(i
ii)

Los componentes
horizontal y vertical de la
fuerza de reacción

∑ 𝐹𝑥 = 0: Es decir,
FMx - FJx = 0, luego
FJx = FMx

(iv)

desarrollada en el
sacro ahora se pueden
CORPORACION UNIVERSITARIA DEL HUILA
CORHUILA

acetábulo de la pelvis (fig. 5.21) forma una articulación


diartrodial. La estabilidad de la articulación de la cadera es
De las ecuaciones (iv) y proporcionada por su
(v):

FJx = 7.4 W
FJy = 7.4 W + W
+W – 0.4 = 9.0 W

Por tanto, la magnitud


de la fuerza resultante
sobre el sacro es:

FJ = √(𝐹𝐽𝑥)2 + (𝐹𝐽𝑦)2
= 11.7 W

Observaciones

• Los resultados
obtenidos son bastante
significativos. Mientras
el atleta está inclinado
hacia adelante en 45° y
levantando un peso con
una magnitud igual a su
propio peso corporal,
los músculos erectores
de la columna ejercen
una fuerza más de 10
veces el peso del atleta
y la fuerza aplicada a la
unión del sacro y el
quinto La vértebra
lumbar es
aproximadamente 12
veces mayor que el
peso corporal.

5.8 Mecánica de la
cadera

La articulación entre la
cabeza del fémur y el
CORPORACION UNIVERSITARIA DEL HUILA
CORHUILA

configuración tipo
balland-socket
relativamente rígida,
sus ligamentos y por
los músculos
grandes y fuertes que
la cruzan. La cabeza
femoral encaja bien
en la cavidad
profunda del
acetábulo. Los
ligamentos de la
articulación de la
cadera, así como el
labrum (un borde
plano de
fibrocartílago),
sostienen y sostienen
la cabeza femoral en
el acetábulo como si
fuera el femoral.
CORPORACION UNIVERSITARIA DEL HUILA
CORHUILA

Fig. 5.21 Pelvis y


cadera: (1) ilion, (2)
sacro, (3) acetábulo, (4)
isquion, (5) trocánter
mayor, (6) trocánter
menor, (7) la cabeza se mueve. La construcción de la articulación de la cadera
fémur

Fig. 5.22 Músculos de


la cadera (a) vistas
anterior y (b)
posterior: (1) psoas,
(2)ilíaco, (3) tensor de la
fascia lata, (4) recto
femoral, (5) sartorio,
(6) gracilis, (7) glúteo
menor, (8) pectíneo, (9)
aductores, (10, 11)
glúteo mayor y medio,
(12) rotadores
laterales, (13) bíceps
femoral, (14)
semitendinoso, (15)
semimembranoso
CORPORACION UNIVERSITARIA DEL HUILA
CORHUILA

es tal que es muy articulaciones sinartrodiales que no permiten ningún


estable y tiene una movimiento. La pelvis se encuentra entre la columna y los dos
gran movilidad, lo fémures. La posición de la pelvis la hace relativamente menos
que permite un estable. Los movimientos de la pelvis ocurren principalmente
amplio rango de con el propósito de facilitar los movimientos de la columna o los
movimiento fémures. No hay músculos cuyo propósito principal sea mover
requerido para la pelvis. Los movimientos de la pelvis son causados por los
actividades como músculos del tronco y la cadera.
caminar, sentarse y
ponerse en cuclillas. Según sus acciones principales, los músculos de la articulación
Los movimientos del de la cadera se pueden dividir en varios grupos (fig. 5.22). El
fémur alrededor de la psoas, el ilíaco, el recto femoral, el pectíneo y el tensor de la
articulación de la fascia lata son los principales flexores de la cadera. También se
cadera incluyen utilizan para realizar actividades como correr o patear. El glúteo
flexión y extensión, mayor y los músculos isquiotibiales (bíceps femoral,
abducción y aducción semitendinoso y semimembranoso) son extensores de la cadera.
y rotación hacia Los músculos isquiotibiales también funcionan como flexores de
adentro y hacia la rodilla. El glúteo medio y el glúteo menor son músculos
afuera. En algunos abductores de la cadera que permiten la rotación del fémur hacia
casos, la extensión de adentro. El glúteo medio es también el grupo de músculos
estos movimientos principal que estabiliza la pelvis en el plano frontal. Los
está restringida por músculos; aductor largo, aductor corto, aductor mayor y gracilis
ligamentos, músculos son los aductores de la cadera. También hay músculos pequeños,
y / o la estructura ósea profundamente colocados (rotadores hacia afuera) que
de la cadera. Las proporcionan la rotación hacia afuera del fémur.
superficies de
articulación de la Los músculos de la cadera sufren predominantemente
cabeza femoral y el contusiones y tensiones que ocurren en la región de la pelvis.
acetábulo están
revestidas con
cartílago hialino. Los
trastornos de la cadera
pueden producir
distribuciones de
fuerza alteradas en el
cartílago articular, lo
que conduce a artritis
degenerativa.

La pelvis consta de los


huesos ilion, isquion,
pubis y sacro. Al
nacer y durante el
crecimiento, los
huesos de la pelvis
son distintos. En los
adultos, los huesos de
la pelvis están
fusionados y forman
CORPORACION UNIVERSITARIA DEL HUILA
CORHUILA

Ejemplo 5.5 Al caminar y correr, colocamos


momentáneamente todo nuestro peso corporal en una pierna (la
pierna derecha en la figura 5.23). Las fuerzas que actúan sobre
la pierna que soporta el peso total del cuerpo se muestran en la
figura 5.24 durante una postura de una sola pierna. FM es la
magnitud de la fuerza resultante ejercida por los músculos
abductores de la cadera, FJ es la magnitud de la fuerza de
reacción articular aplicada

están unidos al fémur, el punto B es el centro de gravedad de la


por la pelvis sobre el pierna y el punto C es donde la fuerza de reacción del suelo es
fémur, W1 es el peso de aplicada en el pie. Las distancias entre el
la pierna, W es el peso
total del cuerpo
aplicado como fuerza
normal por el suelo
sobre la pierna. El
ángulo entre la línea de
acción de la fuerza
muscular resultante y la
horizontal se designa
con θ.

En la figura 5.25 se
muestran un modelo
mecánico de la pierna,
los componentes
rectangulares de las
fuerzas que actúan
sobre ella y los
parámetros necesarios
para definir la
geometría del
problema. O es un
punto a lo largo del eje
instantáneo de rotación
de la articulación de la
cadera, el punto A es
donde los músculos
abductores de la cadera
CORPORACION UNIVERSITARIA DEL HUILA
CORHUILA

punto A y los puntos muestran en términos de sus componentes en las direcciones x


O, B y C se y y. La fuerza muscular
especifican como a,
b y c,
respectivamente. α
es el ángulo de
inclinación del
cuello femoral con
respecto a la
horizontal, y β es el
ángulo que forma el
eje largo de la
diáfisis femoral con
la horizontal. Por lo
tanto, α + β es
aproximadamente
igual al ángulo total
del cuello al eje del
fémur.

Determine la fuerza
ejercida por los
músculos
abductores de la
cadera y la fuerza de
reacción de la
articulación en la
cadera para sostener
la pierna y la cadera
en la posición que se
muestra.

Solución 1: Utilizar
el diagrama de
cuerpo libre de la
pierna. Para
solucionar el
problema, podemos
utilizar el diagrama
de cuerpo libre de la
pierna derecha que
soporta todo el peso
de la persona. En la
figura 5.25a, las
fuerzas de reacción
de los músculos y las
articulaciones se
CORPORACION UNIVERSITARIA DEL HUILA
CORHUILA

Fig. 5.23 Postura de una


pierna

resultante tiene una línea de acción que forma un ángulo θ con la


horizontal. Por lo tanto:

FMx = FM cos θ (i)


FMy = FM sin θ (ii)

Dado que se especifica el ángulo θ (dado como una cantidad


medida), la única incógnita para la fuerza muscular es su magnitud
FM. Para la fuerza de reacción conjunta, no se conocen ni la magnitud
ni la dirección. Con respecto a Con respecto al eje de la articulación
de la cadera ubicada en el punto O, ax en la figura 5.25b es el brazo
de momento de la componente vertical FMy de la fuerza muscular, y
ay es el brazo de momento de la
componente horizontal de la fuerza muscular FMx. De manera
similar,
(bx – ax) es el brazo de momento para W1 y (cx - ax) es el
Fig. 5.24 Fuerzas que
brazo de momento para la fuerza W aplicada por el suelo sobre actúan sobre la pierna la
pierna. derecha que soporta

todo el peso del


De la geometría del problema: cuerpo
ax = a cos a (iii)
ay =
a sin a (iv)
bx =
b cos β (v)
cx = c cos β (vi)
CORPORACION UNIVERSITARIA DEL HUILA
CORHUILA

Fig. 5.25 Diagrama de cuerpo libre del cateto


(a) y los parámetros geométricos (b)
CORPORACION UNIVERSITARIA DEL HUILA
CORHUILA

la cadera es: utilizarse para determinar la magnitud de la fuerza muscular


resultante aplicada en punto A. Suponiendo que los momentos en
el sentido de las agujas del reloj son positivos:

∑ 𝑀𝑂 = 0: axFMy – ayFMx – (cx – ax) W + (bx – ax) W1 = 0

Sustituyendo las ecuaciones (i) a (vi) en la ecuación anterior:

(a cos a) (FM sin θ) – (a sin a) (FM cos θ)


-( c cos β - a cos a)W + (b cos β - a cos a )W1 = 0

Resolviendo esta ecuación para la fuerza muscular:

FM = 𝑐 (𝑊+ 𝑏𝑊1 ) 𝑐𝑜𝑠 𝛽−𝑎 (𝑊− 𝑊1)cos𝑎 =


(vii)

Fig. 5.26
Fuerzas que
actúan sobre la
pelvis durante
una posición de
una sola pierna
(pierna
derecha)

Ahora que se han


establecido las
componentes
horizontal y vertical
de todas las fuerzas
involucradas, y sus
brazos de momento
con respecto al punto
O, la condición para
el equilibrio
rotacional de la
pierna con respecto
al punto O puede
CORPORACION UNIVERSITARIA DEL HUILA
CORHUILA

𝑎 Por lo tanto, la fuerza resultante que actúa en la articulación de


(cos
𝑎
sin FJ = √(𝐹𝐽𝑥)2 + (𝐹𝐽𝑦)2 (x)
𝜃
−sin Suponga que los parámetros geométricos del problema y el peso
𝑎 de la pierna se miden en términos de la altura h y el peso total W
cos𝜃 de la persona de la siguiente manera: a = 0.05h, b= 0.020h, c=
) 0.52h, a= 45°, β = 80°, θ = 70; y W1 = 0.17W. La solución de las
ecuaciones anteriores para las fuerzas de reacción de los
Observe que el músculos y las articulaciones será un ángulo de FM =2.6 FJ =
denominador de la 3.4W, la fuerza de reacción conjunta formando un ángulo φ =
-1
ecuación. (vii) se puede tan (FJy/FJx) = 74.8° con la horizontal.

simplificar como un sin


(θ - α). Para determinar
los componentes de la Solución 2: Utilización del diagrama de cuerpo libre de la parte
fuerza de reacción superior del cuerpo. Aquí tenemos un enfoque alternativo para
conjunta, podemos la solución del mismo problema. En este caso, en lugar del
utilizar las condiciones diagrama de cuerpo libre de la pierna derecha, se utiliza el
de equilibrio horizontal diagrama de cuerpo libre de la parte superior del cuerpo (incluida
y vertical de la pierna: la pierna izquierda). Las fuerzas que actúan sobre la parte
superior del cuerpo se muestran en las Figs. 5.26 y 5.27. FM es la
∑ Fx = magnitud de la fuerza resultante ejercida por los músculos
0: FJx = FMx = FM cos abductores de la cadera aplicados sobre la pelvis en el punto D.
nθ (viii) θ es nuevamente el ángulo entre la línea de acción de la fuerza
∑ Fy = 0: FJy = FMy + muscular resultante y la horizontal. FJ es la magnitud de la fuerza
W – W1 de reacción aplicada por la cabeza del fémur sobre la articulación
de la cadera en el punto E. W2
FJy = FM sin θ + W – = W - W1 (peso corporal total
W1 (ix)
CORPORACION UNIVERSITARIA DEL HUILA
CORHUILA

menos el peso de la pierna derecha) es el peso de la parte superior


del cuerpo y la pierna izquierda actuando como una fuerza
concentrada en el punto G. Tenga en cuenta que el punto G no es
el centro de gravedad de todo el cuerpo. Dado que la pierna
derecha no está incluida en el cuerpo libre, el lado izquierdo del
cuerpo es "más pesado" que el lado derecho, y el punto G está
ubicado a la izquierda del centro de gravedad original (un punto
a lo largo de la línea punteada vertical en la figura 5.27) de la
persona. La ubicación del punto G se puede determinar utilizando
el método proporcionado en la Sección.
4.12.

Combinando los pesos individuales de los segmentos que


constituyen el cuerpo considerado, el problema se reduce a un
sistema de tres fuerzas. De la geometría del problema se
desprende claramente que las fuerzas involucradas no forman un
sistema paralelo. Por lo tanto, para el equilibrio del cuerpo, deben
formar un sistema concurrente de fuerzas. Esto implica que las
líneas de acción de las fuerzas deben tener un punto común de
intersección (punto Q en la figura 5.27), que se puede obtener al Fig. 5.27 Las fuerzas
extender las líneas de acción de W2 y FM. Una línea que pasa por involucradas forman
los puntos Q y E designa la línea de acción de la fuerza de un sistema
reacción conjunta FJ. El ángulo φ que hace FJ con la horizontal concurrente
ahora se puede medir a partir de la geometría del problema. Dado
que la dirección de FJ se determina a través de ciertas
consideraciones geométricas, el número de incógnitas se reduce
en uno. Como se ilustra en la figura 5.28, las magnitudes
desconocidas FM y FJ de las fuerzas de reacción de los músculos
y las articulaciones ahora se pueden determinar simplemente
trasladando W2, FM y FJ al punto Q, y descomponiéndolos en sus
componentes a lo largo de la horizontal (x) y direcciones
verticales (y):

FMx = FM cos θ Fig. 5.28 Resolución de


FMy = FM sin θ las fuerzas en sus
componentes
FJx = FJ cos φ
FJy = FJ sin φ (xi)
Para el equilibrio de traslación en las direcciones x y y:

∑ 𝐹𝑥 = 0 es decir, –FMx + FJx = 0, luego FJx = FMx

∑ 𝐹𝑦 = 0 es decir, FJy – W2 – FMy = 0, luego FJy = FMy +

W2 Considerando la ecuación. (xi):


FJ cos φ = FM cos θ, y (xii)
FJ sin φ = FM sin θ + W2 (xiii)
CORPORACION UNIVERSITARIA DEL HUILA
CORHUILA

De la ecuación (xii):
FJ = 𝐹𝑀 𝑐𝑜𝑠 𝜃
𝑐𝑜𝑠 φ
CORPORACION UNIVERSITARIA DEL HUILA
CORHUILA

Sustituyendo esta ecuación en la ecuación. (xiii) producirá:


M cos θ
F
sin φ = FM sin θ + W2, es
decir, cos φ

F M cos θ sin φ – FM sin


θ = W2 cos φ

FM = ( ) = W2, luego

FM = cos φ 𝑊2, y
Fig. 5.29 Llevar una sin (φ−θ)
carga en cada mano
FJ = cos θ 𝑊2
sin (φ−θ)

Por ejemplo, si θ = 70°, φ = 74.8° y W2 = 0.83W (W es el peso


total de la persona), las dos últimas ecuaciones producirán FM
= 2.6W y FJ = 3.4W.

¿Cómo variarían las fuerzas de reacción de los músculos y

las articulaciones de la cadera si la persona lleva una carga


de W0 en cada mano durante la postura de una sola pierna?
(Fig. 5.29)?
Fig. 5.30 Fuerzas que
En la figura 5.30 se muestra el diagrama de cuerpo libre de la
actúan sobre la parte
parte superior del cuerpo mientras la persona lleva una carga
superior del cuerpo
de W0 en cada mano. El sistema a analizar consiste en la parte
superior del cuerpo de la persona (incluida la pierna izquierda)
y las cargas que lleva en cada mano. Para contrarrestar los
efectos de rotación y traslación (hacia abajo) de las cargas
adicionales, los músculos abductores de la cadera ejercerán
fuerzas adicionales y se generarán fuerzas de compresión
mayores en la articulación de la cadera.

En este caso, el número de fuerzas es cinco. El tirón


gravitacional en la parte superior del cuerpo (W2) y en las
masas llevadas en las manos (W0) forman un sistema de fuerzas
paralelo. Si estas fuerzas paralelas se pueden reemplazar por
una sola fuerza resultante, entonces el número de fuerzas se
Fig. 5.31 W3 es la puede reducir a tres y el problema se puede resolver aplicando
resultante del sistema de la misma técnica explicada anteriormente (Solución 2). Para
tres fuerzas este propósito, considere el sistema de fuerza que se muestra
en la figura 5.31. Los puntos M y N corresponden a las manos
CORPORACION UNIVERSITARIA DEL HUILA
CORHUILA

derecha e izquierda de
la persona donde se
aplican fuerzas
externas de igual
magnitud (W0). El
punto G es el centro de
gravedad de la parte
superior del cuerpo,
incluida la pierna
izquierda. La línea
punteada vertical
muestra el eje de
simetría (línea media)
de la persona en el
plano frontal, y el
punto G está ubicado a
la izquierda de este
eje. Note que la
distancia l 1 entre los
puntos M y G es
mayor que la distancia
l2 entre los puntos N y
G. Si se dan l1, l2, W2
y W0, entonces se
puede determinar un
nuevo centro de
gravedad (punto G’)
aplicando
CORPORACION UNIVERSITARIA DEL HUILA
CORHUILA

la técnica de encontrar el de reacción generada en la articulación de la cadera:


centro de gravedad de un
sistema compuesto por
FM =
varias partes cuyos
cosθcos sin φ φ′𝑊′−2𝑠𝑖
centros de gravedad son
conocidos (véase la 𝑛+2
sección 5.14). Por
intuición, el punto G’ está
ubicado en algún lugar
entre el eje de simetría y el
punto
G. En otras palabras, G’
está más cerca de la
articulación de la cadera
derecha y, por lo tanto, de
la longitud del brazo de
momento.

del peso total medido


desde la articulación de
la cadera derecha es más
corto en comparación
con el caso cuando no se
lleva carga en las manos.
Por otro lado, la
magnitud de la fuerza
gravitacional resultante
es W3 = W2 + 2W0, que
sobre compensa la
ventaja obtenida por la
reducción del brazo de
momento.

Una vez que se determina


el nuevo centro de
gravedad de la parte
superior del cuerpo,
incluida la pierna
izquierda y las cargas
llevadas en cada mano,
las ecuaciones (xi) y (xii)
se pueden utilizar para
calcular la fuerza
resultante ejercida por
los músculos abductores
de la cadera y la fuerza
CORPORACION UNIVERSITARIA DEL HUILA
CORHUILA

θ𝑊cos Suponiendo que el sistema que estamos analizando consiste


0 φ′ en la parte superior del cuerpo, la pierna izquierda y la carga
en la mano, la carga adicional W0 llevada en la mano
izquierda desplazará el centro de gravedad del sistema del
FJ = punto G al punto G’’ hacia la izquierda de la persona. En
cosθco consecuencia, aumentará la longitud del brazo de palanca de
s sin θ la fuerza gravitacional total W4 =W2 + W0 medida desde la
φ(𝑊′− articulación de la cadera derecha (figura 5.34). Esto requerirá
2+𝑠𝑖𝑛2 mayores fuerzas del músculo abductor de la cadera para
𝑊θcos contrarrestar el efecto de rotación en el sentido de las agujas
0) φ′ del reloj de W4 y también aumentar las fuerzas de compresión
en la articulación de la cadera derecha.
Aquí, en las ecuaciones
(xi) y (xii) se modifican Se puede observar a partir de la geometría del sistema
reemplazando el peso analizado que un desplazamiento del centro de gravedad del
W2 de la parte superior punto G al punto G’’ hacia la izquierda de la persona
del cuerpo con el nuevo disminuirá el ángulo entre la línea de acción de la fuerza de
peso total W3 = W2 + reacción articular y la horizontal de φ hasta φ’’. Para la nueva
2W0, y reemplazando el configuración del cuerpo libre que se muestra en la figura
ángulo φ que forma la 5.34, las ecuaciones (xi) y (xii) se pueden utilizar nuevamente
línea de acción de la para calcular la fuerza requerida del músculo abductor de la
fuerza de reacción cadera y la fuerza de reacción articular producida en la cadera
conjunta con la derecha (opuesto al lado donde se lleva la carga):
horizontal con el nuevo
ángulo φ’ (Fig. 5.32). FM = cosθcos sin φ φ′′′′(𝑊−𝑠𝑖𝑛2+ θ𝑊cos0)
φ’ es ligeramente más
φ′′
grande que φ debido al
desplazamiento del
centro de gravedad del
FJ = cosθ cossin φθ(′′𝑊−2𝑠𝑖𝑛+ 𝑊θ0cos) φ′′
punto G al punto G’
hacia la derecha de la
persona. Además, se
asume que el ángulo θ
entre la línea de acción
de la fuerza muscular y
la horizontal
permanece sin
cambios.

¿Qué sucede si la
persona lleva una
carga de W0 en la mano
izquierda durante la
posición de la pierna
derecha? (figura 5.33)
CORPORACION UNIVERSITARIA DEL HUILA
CORHUILA

Fig. 5.32 El problema se


reduce a un sistema
concurrente de tres
fuerzas

Fig. 5.33 Llevar una


carga en una mano
CORPORACION UNIVERSITARIA DEL HUILA
CORHUILA

Fig. 5.35 La rodilla: (1) fémur, (2) cóndilo medial, (3) cóndilo
lateral, (4) menisco medial, (5) menisco lateral, (6) ligamento
colateral tibial, (7) ligamento colateral peroneo, (8) tibia, (9)
peroné, (10) tendón del cuádriceps, (11) rótula,
Fig. 5.34 Fuerzas que (12) ligamento rotuliano
actúan sobre el parte
superior del cuerpo
Observaciones

• Cuando el peso corporal se apoya por igual en ambos


pies, la mitad del peso supra femoral cae sobre cada
CORPORACION UNIVERSITARIA DEL HUILA
CORHUILA

articulación de la cadera. músculos abductores de la cadera débiles y / o articulaciones de la


Al caminar y correr, toda cadera dolorosas generalmente se inclinan hacia el lado más débil
la masa del cuerpo es y caminan con el llamado paso abductor. Inclinar el tronco
sostenida lateralmente hacia la cadera afectada desplaza
momentáneamente por
una articulación, y hemos
analizado algunos de
estos casos.

• Los análisis
anteriores indican que las
fuerzas de apoyo
necesarias en la
articulación de la cadera
son mayores cuando se
lleva una carga en el lado
opuesto del cuerpo en
comparación con las
fuerzas necesarias para
llevar la carga cuando se
distribuye a ambos lados.
Transportar cargas con
ambas manos y
acercándolas a la línea
media del cuerpo es eficaz
para reducir las fuerzas
musculo esqueléticas
necesarias.

• Al llevar una carga de


un lado, las personas

tienden a inclinarse hacia


el otro lado. Esto acerca el
centro de gravedad de la
parte superior del cuerpo
y la carga que se lleva en
la mano a la línea media
del cuerpo, reduciendo así
la longitud del brazo de
momento de la fuerza
gravitacional resultante
medida desde la
articulación de la cadera
distal a la carga.

• Las personas con


CORPORACION UNIVERSITARIA DEL HUILA
CORHUILA

el centro de gravedad para soportar grandes cargas. Es un componente esencial del


del cuerpo más cerca sistema de enlace responsable de la locomoción humana. La
de la articulación de la rodilla es extremadamente vulnerable a las lesiones.
cadera y, en
consecuencia, reduce La rodilla es una estructura de dos articulaciones compuesta por
la acción de rotación la articulación tibiofemoral y la articulación femororrotuliana
del momento del peso (fig. 5.35). La articulación tibiofemoral tiene dos articulaciones
corporal sobre la distintas entre los cóndilos medial y lateral del fémur y
articulación de la Cóndilos laterales del fémur y la tibia. Estas articulaciones
cadera al reducir su
brazo de momento. A
cambio, esto reduce la
magnitud de las están separadas por capas de cartílago, llamadas meniscos.
fuerzas ejercidas por Los meniscos lateral y medial eliminan el contacto de hueso a
los músculos hueso entre el fémur y la tibia y funcionan como
abductores de la amortiguadores. La articulación femororrotuliana es la
cadera necesarios para articulación entre la rótula y
estabilizar la pelvis. el extremo anterior de los cóndilos femorales. La rótula es un
hueso "flotante" que el tendón del cuádriceps y el ligamento
• La marcha del rotuliano mantienen en posición. Aumenta la ventaja
abductor se puede mecánica del músculo cuádriceps, mejorando su efecto de
corregir de manera tracción sobre la tibia a través del tendón rotuliano. La
más eficaz con un estabilidad de la rodilla es proporcionada por una intrincada
bastón en la mano estructura ligamentosa, los meniscos y los músculos que
opuesta a la cadera cruzan la articulación. La mayoría de las lesiones de rodilla se
débil, en comparación caracterizan por daños en los ligamentos y cartílagos que se
con el bastón producen en el lado medial.
sostenido en la mano
del mismo lado que la Los músculos que cruzan la rodilla la protegen, proporcionan
cadera débil. fuerzas internas para el movimiento y / o controlan su
movimiento. El control muscular de la rodilla lo producen
5.9 Mecánica de la principalmente los músculos cuádriceps y el grupo de
rodilla músculos isquiotibiales (fig. 5.36). El grupo de músculos
cuádriceps está compuesto por los músculos recto femoral,
La rodilla es la vasto lateral, vasto medial y vasto intermedio. El músculo
articulación más recto femoral tiene inserciones en la espina ilíaca
grande del cuerpo. Es anteroinferior y la rótula, y sus acciones principales son la
una articulación de
bisagra modificada.
Además de la acción
de flexión y extensión
de la pierna en el plano
sagital, la articulación
de la rodilla permite
cierta rotación
automática hacia
adentro y hacia afuera.
La articulación de la
rodilla está diseñada
CORPORACION UNIVERSITARIA DEL HUILA
CORHUILA

flexión de la cadera y la para fortalecer los músculos del cuádriceps (figura 5.37).
extensión de la rodilla.
Los músculos vasto Las fuerzas que actúan sobre la pierna y un modelo mecánico
lateral, medial e simple de la pierna se ilustran en la figura 5.38. W1 es el
intermedio conectan el
fémur y la tibia a través
de la rótula, y todos son
extensores de la rodilla.
Los músculos bíceps
femoral, semitendinoso
y semimembranoso
forman el grupo de
músculos isquiotibiales,
que ayudan a controlar la
extensión de la cadera, la
flexión de la rodilla y
cierta rotación hacia
adentro y hacia afuera de
la tibia. Los músculos
semitendinoso y
semimembranoso tienen
uniones proximales en el
hueso pélvico y uniones
distales en la tibia. El
bíceps femoral tiene
inserciones proximales
en el hueso pélvico y el
fémur, y las inserciones
distales en el peroné.
También está el músculo
poplíteo que tiene
inserciones en el fémur y
la tibia. La función
principal de este músculo
es la flexión de la rodilla.
Los otros músculos de la
rodilla son sartorios,
gracilis, gastrocnemio y
plantar.

Ejemplo 5.6 Considere


una persona que usa una
bota de pesas y, sentada,
realiza ejercicios de
flexión / extensión de la
parte inferior de la pierna
CORPORACION UNIVERSITARIA DEL HUILA
CORHUILA

peso de la parte
inferior de la pierna,
W0 es el peso de la
bota, FM es la
magnitud de la
fuerza de tracción
ejercida por el
músculo cuádriceps
sobre la tibia a través
del tendón rotuliano
y FJ es la magnitud de
la fuerza de reacción
de la articulación
tibiofemoral aplicada
por el fémur en la
meseta tibial. El
centro de la
articulación
tibiofemoral se
encuentra en el punto
O, el tendón
rotuliano está unido a la
tibia en el punto A, el Fig. 5.37 Ejercicio de los músculos alrededor de la
centro
articulación de la
rodilla

Fig. 5.36 Músculos de la


rodilla: (1) recto
femoral, (2) vasto
medial, (3) vasto
intermedio, (4) vasto
lateral, (5) ligamento
rotuliano, (6)
semitendinoso, (7)
semimembranoso, (8)
bíceps
femoral, (9)
gastrocnemio
CORPORACION UNIVERSITARIA DEL HUILA
CORHUILA

La gravedad de la parte inferior de la pierna se encuentra en el


punto B, y el centro de gravedad de la bota de pesas está ubicado
en el punto C. Las distancias entre el punto O y los puntos A, B y
C se miden como a, b y c, respectivamente. Para la posición de la
parte inferior de la pierna que se muestra, el eje largo de la tibia
forma un ángulo β con la horizontal, y la línea de acción de la
fuerza del músculo cuádriceps forma un ángulo θ con el eje largo
de la tibia.

Suponiendo que los puntos O, A, B y C están todos a lo largo de


una línea recta, determine FM y FJ en términos de a, b . c, θ, β, W1
y W0.
Fig. 5.38 Fuerzas que
actúan sobre la pierna

Solución: La componente horizontal (x) y vertical (y) de las


fuerzas que actúan sobre la pierna y sus brazos de palanca,
medidas desde la articulación de la rodilla ubicada en el punto O,
se muestran en la figura 5.39. Los componentes de la fuerza
muscular son:

FMx = FM cos (θ + β) (i)


FMy = FM sin (θ + β) (ii)

Hay tres incógnitas, a saber, FM, FJx y FJy. Para la solución de este
problema bidimensional (plano), deben utilizarse las tres
condiciones de equilibrio. Suponiendo que los momentos en
Fig. 5.39 sentido contrario a las agujas del reloj son positivos, considere el
Componentes de equilibrio rotacional de la parte inferior de la pierna con respecto
fuerza y sus brazos de al punto O:
palanca

∑ Mo = 0: (a cos β) FMy – (a sin β)FMx


- (b cos β) W1 - (c cos β) W0 = 0

Sustituyendo las ecuaciones (i) y (ii) en la ecuación anterior, y


resolverlo para FM dará como resultado:
CORPORACION UNIVERSITARIA DEL HUILA
CORHUILA

FM = (𝑏𝑊1+𝑐 𝑊0) 𝑐𝑜𝑠𝛽


(iii)
a[cos𝛽 sin (𝜃 + 𝛽)−𝑠𝑖𝑛𝛽cos (𝜃
+ 𝛽)]

Tenga en cuenta que


esta ecuación se
puede simplificar
considerando que [
cos β sin (θ + β) - sin
β cos sin (θ + β)] = sin
θ; es decir:
CORPORACION UNIVERSITARIA DEL HUILA
CORHUILA

= 22 cm, c = 50 cm, W1 = 150N, W0 =100 N, θ =15°; y β = 45°,


luego usando ecuaciones (iii) y (iv):
FM =
(𝑏𝑊1 FM = 1853 N, FJ = 1681 N, φ = 55.7°
+𝑐

𝑊0)
𝑐𝑜𝑠𝛽
a sin 𝜃 Observaciones

La ecuación (iii) arroja • La fuerza FM ejercida por el músculo cuádriceps sobre la


la magnitud de la tibia a través del tendón rotuliano se puede expresar en términos
fuerza que deben de dos componentes normales y tangenciales al eje largo de la
ejercer los músculos tibia (fig. 5.40). La función principal del componente normal FMn

cuádriceps para a lo largo de las direcciones horizontal y vertical también se


sostener la pierna pueden evaluar considerando el equilibrio de traslación de la
parte inferior de la pierna en las direcciones x y y:
cuando se extiende
hacia adelante
∑ 𝐹𝑥= 0: FJx = FMx = FM cos (θ + β)
formando un ángulo β
con la horizontal. Una ∑ 𝐹𝑦= 0: FJy = FMy – W0 – W1
vez que se determina la FJy = FM sin (θ + β) - W0 – W1
FM, los componentes
de la fuerza de reacción
desarrollada en la
articulación de la de la fuerza muscular es rotar la tibia alrededor de la articulación
rodilla La magnitud de de la rodilla, mientras que su componente tangencial FMt tiende a
la fuerza de trasladar la parte inferior de la pierna en una dirección colineal
compresión resultante con el eje largo de la tibia y aplica una fuerza de compresión en
aplicada sobre la las superficies de articulación de la articulación tibiofemoral.
meseta tibial en la Dado que el componente normal de FM es una función sinusoidal
articulación de la del ángulo θ, un ángulo más grande entre el tendón rotuliano y el
rodilla es: eje largo de la tibia indica un efecto de rotación mayor del
esfuerzo muscular. Esto implica que para θ grandes, se
desperdicia menos fuerza muscular para comprimir la
articulación de la rodilla y se utiliza una porción mayor de la
FJ = √(𝐹𝐽𝑥)2 + (𝐹𝐽𝑦)2 (iv) tensión muscular para rotar la parte inferior de la pierna alrededor
de la articulación de la rodilla.
φ
• Una de las funciones biomecánicas más importantes de la
= arc tan ( 𝐹 rótula es proporcionar el desplazamiento anterior de los tendones
𝐹𝐽𝑥 del cuádriceps y de la rótula, alargando así el brazo de palanca de
las fuerzas del músculo extensor de la rodilla con respecto al
Suponga que los centro de rotación de la rodilla aumentando el ángulo θ (Fig.
parámetros 5.41a). La extirpación quirúrgica de la rótula acerca el tendón
geométricos y los rotuliano al centro de rotación de la articulación de la rodilla (fig.
pesos involucrados se 5.41b), lo que hace que la longitud del brazo de palanca de la
dan como: a = 12 cm, b fuerza muscular disminuya (d2 <d1). Al perder la ventaja de tener
un brazo de palanca relativamente largo, el músculo cuádriceps
CORPORACION UNIVERSITARIA DEL HUILA
CORHUILA

tiene que ejercer más


fuerza de lo normal
para rotar la parte
inferior de la pierna
alrededor de la
articulación de la
rodilla.

• La rodilla
humana tiene una
estructura de dos
articulaciones
compuesta por
articulaciones
tibiofemoral y
patelofemoral.
Observe que el
músculo cuádriceps
pasa por encima de la
rótula, y la rótula y el
músculo forman una
disposición de cuerda
de polea. Cuanto
mayor es la tensión
en el músculo, mayor
es la fuerza de
compresión (presión)
que ejerce la rótula
sobre la articulación
femororrotuliana
CORPORACION UNIVERSITARIA DEL HUILA
CORHUILA

Fig. 5.40 Componentes rotacionales y de


traducción de FM

Fig. 5.41 La rótula aumenta la longitud del brazo nivelado

Fig. 5.42 Análisis estático de las fuerzas que actúan sobre la


CORPORACION UNIVERSITARIA DEL HUILA
CORHUILA

rótula Hemos analizado las fuerzas involucradas alrededor de la


articulación tibiofemoral considerando el diagrama de cuerpo
libre de la parte inferior de la pierna. Una vez determinada la
tensión en el tendón rotuliano y suponiendo que la tensión es
uniforme en todo el cuádriceps, podemos calcular la fuerza de
compresión aplicada sobre la articulación femororrotuliana
considerando el diagrama de cuerpo libre de la rótula (fig.
5.42). Sea FM la magnitud uniforme de la fuerza de tracción
en los tendones rotuliano y cuádriceps, FP sea la magnitud de
la fuerza ejercida sobre la articulación femororrotuliana, α sea
CORPORACION UNIVERSITARIA DEL HUILA
CORHUILA

el ángulo entre el tendón


rotuliano y la horizontal, γ Estas ecuaciones se pueden resolver simultáneamente para el
sea el ángulo entre el ángulo φ y la magnitud FP de la fuerza de compresión aplicada
tendón del cuádriceps y la por el fémur sobre la rótula en la articulación
horizontal, y φ es el femororrotuliana:
ángulo desconocido entre
la línea de acción de la De la ecuación (v):
fuerza de reacción de
compresión en la FP = 𝐹𝑀 (cos 𝛾 – 𝑐𝑜𝑠 𝛾)
articulación (figura cos φ

Sin φ (cos 𝛾 – 𝑐𝑜𝑠 𝛾)= cos φ (sin 𝛾 − 𝑠𝑖𝑛 𝛾),

tan φ = sin 𝛾 − 𝑠𝑖𝑛 𝛾


y cos 𝛾 –
𝑐𝑜𝑠 𝛾
φ = tan- ( sin 𝛾 − 𝑠𝑖𝑛 𝛾)
1

cos 𝛾 – 𝑐𝑜𝑠 𝛾

aplicar las ecuaciones de equilibrio. Para el equilibrio de la rótula


5.42b) y la horizontal. en las direcciones x y y:
Tenemos un sistema de
tres fuerzas y para el
equilibrio de la rótula ∑ 𝐹𝑥= 0: FP cos φ = FM = (cos γ – cos α) (v)
tiene que ser concurrente. ∑ 𝐹𝑦= 0: FP sin φ = FM (sin α - sin γ ) (vi)

Primero podemos
determinar el punto
común de intersección Q
extendiendo las líneas de
acción de las fuerzas del
tendón rotuliano y
cuádriceps. Una línea que
conecta el punto Q y el
punto de aplicación de FP
corresponderá a la línea
de acción de FP. Luego,
las fuerzas se pueden
trasladar al punto Q
(figura 5.42c) y se pueden
CORPORACION UNIVERSITARIA DEL HUILA
CORHUILA

estabilidad.

De la ecuación (vi): El complejo de la articulación del tobillo consta de las


articulaciones tibiotalar, fibulotalar y tibiofibular distal. La
FP = 𝐹𝑀 articulación del tobillo (tibiotalar) es una bisagra o
(sin 𝛾 − articulación tipo ginglymus entre la superficie convexa en
𝑠𝑖𝑛 𝛾), es
decir sin φ

𝐹 𝑀 (cos 𝛾
– 𝑐𝑜𝑠 𝛾) = 𝐹𝑀
(sin 𝛾 − 𝑠𝑖𝑛 𝛾) , luego cos φ sin φ
Una vez que se
determina el ángulo φ,
también se puede
determinar la
magnitud de la fuerza
ejercida sobre la
articulación
femororrotuliana FP:

FP = FM ( cos 𝛾 – 𝑐𝑜𝑠 𝛾)
𝑐𝑜𝑠 φ

5.10 Mecánica del


tobillo

El tobillo es la unión
de tres huesos: la tibia,
el peroné y el astrágalo
del pie (fig. 5.43). Al
igual que otras
articulaciones
importantes de la
extremidad inferior, el
tobillo es responsable
de las funciones
cinemáticas y de
carga. La articulación
del tobillo es
inherentemente más
estable que la
articulación de la
rodilla, lo que requiere
restricciones
ligamentosas y
musculares para su
CORPORACION UNIVERSITARIA DEL HUILA
CORHUILA

forma de carrete de la como el sóleo se encuentran en el compartimento posterior de la


tróclea del astrágalo y el pierna y se insertan en la superficie posterior del calcáneo a
extremo distal cóncavo de través del tendón de Aquiles. El gastrocnemio cruza las
la tibia. Al ser una articulaciones de la rodilla y el tobillo y tiene
articulación en bisagra, el
tobillo sólo permite el
movimiento de flexión-
extensión (dorsiflexión-
flexión plantar) del pie en
el plano sagital. Otros
movimientos del pie
incluyen inversión y
eversión, rotación hacia
adentro y hacia afuera y
pronación y supinación.
Estos movimientos
ocurren alrededor de las
articulaciones del pie,
como la articulación
subastragalina entre el
astrágalo y el calcáneo y
las articulaciones
transversales del tarso,
talonavicular y calcáneo-
cuboide.

La mortaja del tobillo se


mantiene mediante la
forma de las tres
articulaciones y los
ligamentos y músculos
que cruzan la
articulación. La
integridad de la
articulación del tobillo
mejora mediante los
sistemas de ligamentos
colaterales medial
(deltoides) y lateral, y los
ligamentos interóseos.
Hay numerosos grupos de
músculos que cruzan el
tobillo. Los flexores
plantares del tobillo más
importantes son los
músculos gastrocnemio y
sóleo (fig. 5.44). Tanto el
músculo gastrocnemio
CORPORACION UNIVERSITARIA DEL HUILA
CORHUILA

funciones en ambos.
En la rodilla colabora
con la flexión de la
rodilla y en el tobillo
es el principal flexor
plantar. Los
extensores plantares o
dorsiflexores son
músculos anteriores.
Son los músculos,
tibial anterior,
extensor largo de los
dedos, extensor largo
del dedo gordo y
peroneo tercio. La
función principal de
los músculos laterales
(el peroneo largo y el
peroneo corto) es
ejercer y flexionar
plantar el tobillo. Fig. 5.44 Músculos del tobillo (a) vista posterior, (b) anterior
y
La articulación del (c) lateral: (1) gastrocnemio, (2) sóleo, (3) tendón de Aquiles,
tobillo responde mal a (4) tibial anterior, (5) extensor largo de los dedos , (6)
pequeños cambios en extensor largo del dedo gordo, (7) peroneo largo, (8) peroneo
su configuración corto
anatómica. La pérdida
de restricciones
cinemáticas y
estructurales debido a
esguinces severos
puede afectar
seriamente el tobillo

Fig. 5.43 El tobillo y el


pie: (1) tibia, (2) peroné,
(3) maléolo
medial, (4) maléolo
lateral, (5) astrágalo,
(6) calcáneo
CORPORACION UNIVERSITARIA DEL HUILA
CORHUILA

estabilidad y puede producir una mala alineación de las


superficies de la articulación del tobillo. La lesión de tobillo
más común, el esguince por inversión, ocurre cuando el peso
del cuerpo se transmite con fuerza al tobillo mientras el pie
está invertido (la planta del pie hacia adentro).

Fig. 5.45 Las fuerzas que Ejemplo 5.7 Considere a una persona que está de puntillas
actúan sobre el pie forman sobre un pie (se ilustra una posición extenuante). Las fuerzas
un sistema concurrente de que actúan sobre el pie durante este instante se muestran en
fuerzas la figura 5.45. W es el peso de la persona aplicado sobre el
pie como la fuerza de reacción del suelo, FM es la magnitud
de la fuerza de tracción ejercida por los músculos
gastrocnemio y sóleo en el calcáneo a través del tendón de
Aquiles, y FJ es la magnitud de la fuerza de reacción aplicada
en la articulación del tobillo por la tibia en la cúpula del
astrágalo. El peso del pie es pequeño en comparación con el
peso del cuerpo y, por lo tanto, se ignora. El tendón de
Aquiles está unido al calcáneo en el punto A, el centro de la
articulación del tobillo está ubicado en el punto B y la fuerza
de reacción del suelo se aplica sobre el pie en el punto C.
Para esta posición del pie, se estima que la línea de La acción
de la fuerza de tracción en el tendón de Aquiles forma un
ángulo θ con la horizontal, y la línea de acción de la fuerza
de reacción de la articulación del tobillo forma un ángulo β
con la horizontal.
Fig. 5.46 Componentes de
las fuerzas que actúan Suponiendo que se conocen las posiciones relativas de los
sobre el pie puntos A, B y C, determine las expresiones de la tensión en
el tendón de Aquiles y la magnitud de la fuerza de reacción
en la articulación del tobillo.

Solución: Tenemos un sistema de tres fuerzas compuesto


por la fuerza muscular FM, la fuerza de reacción de la
articulación FJ y la fuerza de reacción del suelo W. De la
CORPORACION UNIVERSITARIA DEL HUILA
CORHUILA

geometría del problema,


es obvio que para la
posición del pie que se
muestra, las fuerzas que
actúan sobre el pie no
forma un sistema de
fuerza paralelo. Por
tanto, el sistema de
fuerzas debe ser
concurrente. El punto
común de intersección
(punto O en la figura
5.45) de estas fuerzas se
puede determinar
extendiendo las líneas
de acción de W y
CORPORACION UNIVERSITARIA DEL HUILA
CORHUILA

FM. Una línea recta que línea de acción de la fuerza de reacción conjunta.
pasa por los puntos O y B
representa la línea de Una vez que se determina la línea de acción de la fuerza de
acción de la fuerza de reacción de la articulación por medios gráficos, las
reacción conjunta. magnitudes de la reacción de la articulación y las fuerzas
Suponiendo que se conocen musculares se pueden calcular traduciendo las tres fuerzas
las posiciones relativas de involucradas al punto común de intersección en O (figura
los puntos A, B y C (como 5.46). Las dos incógnitas FM y FJ ahora se pueden determinar
se indica en el problema), aplicando las condiciones de equilibrio de traslación en las
se puede medir el ángulo direcciones horizontal (x) y vertical (y).
(digamos, β) de la
Para ello, la reacción conjunta

y las fuerzas musculares deben descomponerse en sus componentes


rectangulares primero:
FMx = FM cos θ
FMy = FM sin θ
FJx = FJ cos β
FJy = FJ sin β

y las fuerzas musculares deben descomponerse en sus


componentes rectangulares primero:

∑ 𝐹𝑥= 0: FJx = FMx, es decir, FJ cos β = FM cos θ

∑ 𝐹𝑦= 0: FJy = FMy + W, es decir, FJ sin β = FM sin θ

Las soluciones simultáneas de estas ecuaciones producirán:

FM = W 𝑐𝑜𝑠 𝛽 es decir: FM = 𝑤 𝑐𝑜𝑠 𝛽 cosθ sin


𝛽−𝑠𝑖𝑛θcos𝛽´ 𝑠𝑖𝑛 (𝛽−𝜃)
FJ = W 𝑐𝑜𝑠 𝜃 es decir: FJ = 𝑤 𝑐𝑜𝑠 𝜃 cosθ sin
𝛽−𝑠𝑖𝑛θcos𝛽´ 𝑠𝑖𝑛 (𝛽−𝜃)

Por ejemplo, suponga que θ = 45° y β = 60°. Entonces:

FM = 1.93 FJ = 2.73 W

5.11 Problemas de Ejercicio


CORPORACION UNIVERSITARIA DEL HUILA
CORHUILA

Problema 5.1 Considere una persona que sostiene un objeto en la


mano con el codo flexionado en ángulo recto con respecto a la
parte superior del brazo (figura 5.4). Las fuerzas que actúan sobre
el antebrazo y el modelo mecánico del sistema se muestran en la
figura 5.5a, b. En cuanto a este sistema, suponga que el bíceps es
el flexor principal y que la línea de acción del músculo forma el
ángulo recto con el eje largo del antebrazo. El punto O designa el
eje de rotación en la articulación del codo, A es el punto de unión
del músculo bíceps al radio, el punto B es el centro de gravedad
del antebrazo y el punto C es el centro de gravedad del objeto
CORPORACION UNIVERSITARIA DEL HUILA
CORHUILA

sostenido en la mano. Además, las distancias entre el eje de


rotación de la articulación del codo (punto O) y los puntos A, B y
C son a = 4.5 cm, b = 16.5 cm y c = 37 cm. Si el peso total del
antebrazo es W = 83 N, y la magnitud de la fuerza muscular es FM
= 780 N:

(a) Determine el peso (W0) del objeto que se sostiene en la mano.

(b) Determine la magnitud de la fuerza de reacción (FJ) en la


articulación del codo.

(c) Determine la magnitud de las fuerzas de reacción del músculo


(FM1) y de la articulación (FJ1) cuando el peso del objeto
sostenido en la mano aumenta en 5 N.

Respuestas: (a) WO = 57.8 N; (b) FJ = 639.2 N; (c) = FM1 = 820 N,


FJ1 = 674.2 N

También podría gustarte